0 of 310 Questions completed
Questions:
You have already completed the quiz before. Hence you can not start it again.
Quiz is loading…
You must sign in or sign up to start the quiz.
You must first complete the following:
0 of 310 Questions answered correctly
Your time:
Time has elapsed
You have reached 0 of 0 point(s), (0)
Earned Point(s): 0 of 0, (0)
0 Essay(s) Pending (Possible Point(s): 0)
Average score |
|
Your score |
|
A 62-year-old male was admitted with sudden onset dizziness and slurred speech. He has a past medical history of hypertension, BPPV, epilepsy and episodic migraine with aura. His medications include lisinopril, atorvastatin, carbamazepine and flunarazine.
On examination, there is evidence of left-sided appendicular ataxia and hemifacial sensory loss with contralateral hemisensory spinothalamic loss below the face. His left pupil is 3mm and right pupil is 5mm. Power examination is normal.
What is the most likely cause of his symptoms?
A 44-year-old man presents to the Emergency Department with progressive confusion, skin changes and diarrhoea. He has a past history of alcoholism, recent bariatric surgery and has type 2 diabetes mellitus. His medications include insulin, metformin and atorvastatin.
On examination, you notice a sunburn-like rash over sun-exposed areas including his hands and his neck – the latter consistent with Casal’s necklace. His GCS is 14 (M5V5E4) and there are no lateralising neurological signs.
You commence pabrinex and monitor for alcohol withdrawal using local hospital protocols. Brain imaging is unrevealing.
What is the most likely underlying cause?
A 62-year-old female presents to the Emergency department with sudden onset rotary vertigo, left-sided hypoacusis, hemiataxia and hemifacial sensory loss with loss of pain and temperature sensation affecting the right arm and leg.
On examination, she has a Horner’s syndrome on the left
Which of the following is the most likely diagnosis?
A 54-year-old woman presented to the Neurology Outpatient Clinic with a 6-month history of episodes of excruciating right-sided facial pain. The episodes are brief and described as an “electric shock” across the right side of her face. The episodes are frequently triggered by brushing her teeth and applying makeup. Her GP has prescribed Carbamazepine with limited benefit.
An MRI scan of her brain with trigeminal root entry zone sequences was performed, which demonstrated contact between the right superior cerebellar artery (SCA) and the right trigeminal nerve.
What is the next most appropriate management strategy?
A 45-year-old male is admitted with an acute total anterior circulation stroke. There is a family history of young strokes on his father’s side.
Diagnostic work-up with carotid dopplers, trans-thoracic echocardiogram and serological tests are unremarkable.
An MRI-brain reveals white matter signal change within the anterior temporal poles bilaterally with dilated perivascular spaces and subcortical lacunes. A skin biopsy is done given diagnostic suspicion, which reveals granular osmophilic material in the small arterioles on electron microscopic examination.
Given the likely diagnosis, which gene mutation is most likely to be identified?
A 28-year-old woman with generalised epilepsy is contacted by Public Health England when her housemate is diagnosed with Neisseria meningitis. They advise her to start a course of prophylactic rifampicin. Her seizures are usually well controlled with a combination of carbamazepine and lamotrigine. Her other medications are folic acid, vitamin D supplements and the combined oral contraceptive pill.
Which of the following would you advise?
A 62-year-old male is admitted to a District General Hospital with sudden onset right-sided weakness and expressive dysphasia. His symptoms started 4 hours ago. He has a past medical history of atrial fibrillation, hypertension and a previous stroke 4 months ago. His medications include amlodipine and apixaban, which his wife reports he has not taken for two days ago.
On examination he has a dense right-sided hemiplegia, facial droop with forehead sparing, expressive dysphasia and right homonymous hemianopia. His National Institute of Health Stroke Scale score (NIHSS) is 18. His observations reveal a heart rate of 92bpm, respiratory rate 18/min, blood pressure is 210/111mmHg and he is afebrile. An ECG reveals atrial fibrillation. Labetolol is administered with his blood pressure reducing to 190/108mmHg.
A CT-brain is performed, which is normal, and a CT-angiogram confirms a left M2 occlusion.
You are planning on thrombolysis and discussing with the regional thrombectomy team. Which of the following is a contraindication to thrombolysis in this patient?
What is the most common cause of an abducens nerve palsy?
A 20-year-old female presented with new-onset status refractory status epilepticus. On the day of admission, she complained of headache and vomited.
She had a history of low mood and 2 days previously had been treated for paracetamol overdose and discharged.
Examination off sedation revealed complex ophthalmoplegia and bilateral ptosis.
CSF examination showed WCC 1/mm3 (negative gram stain and negative standard viral PCR), protein 0.4g/L, glucose 4.1 mmol/mol.
Axial T2 (A) and axial diffusion-weighted (B) MRI are below.
What is the most likely diagnosis?
A 58-year-old female with a past medical history of migraine and hypertension presented to the hospital with a two-week history of headache and nausea. She described a frontal band-like headache.
Her neurological examination was normal and blood pressure 145/85mmHg. She is afebrile. Kernig and Brudzinski signs are negative and there is no papilloedema.
Her neuroimaging is shown below (left = T2 coronal, right = T1 coronal with gadolinium).
What is the likely cause of her headaches?
A 20-year-old female with a longstanding history of fluctuating predominantly lower limb dystonia and walking difficulties since childhood has a dramatic improvement after taking levodopa.
Which of the following statements is true of her diagnosis?
A patient attends your clinic with a complaint of intermittent vertigo, exacerbated by movement. You suspect otoconia in the left horizontal semicircular canal.
You perform a modified Dix-Hallpike manoeuvre (supine roll test).
Which of the following would confirm your suspicions?
A 5-year-old child with a history of a single febrile seizure at 26 months old is brought to the Accident and Emergency Department by his parents following an episode at home earlier in the night. His father describes the boy waking several hours after going to bed and complaining of nausea. He then vomited and became pale and sweaty. About 10 minutes later, his eyes deviated to the left, the right side of his face began twitching and his right arm jerked for around one minute. Over the next 20 minutes he remained pale, sweaty and confused.
When you review the boy nearly three hours after onset, his observations are all in normal ranges. He is alert and appropriate but cannot remember the event.
On further questioning, he had a similar event 4 months ago, during which he vomited, became pale and sweaty with a “racing pulse” and developed twitching of the left face and arm.
You suspect Panayiotopoulos syndrome. Which of the following is true of this disorder?
An elderly gentleman presents to the hospital following an unwitnessed fall with severe headache, vomiting, and altered level of consciousness.
On examination, the patient has a dilated and unresponsive right pupil, and left-sided hemiparesis.
What herniation syndrome is this?
A 34-year-old female is seen in outpatient neurology follow-up accompanied by her husband. She was admitted 6 weeks ago with headaches and fevers and had been started on treatment for confirmed herpes simplex virus 1 encephalitis (HSVE). She made a good recovery 3 weeks following her illness onset and was discharged with early follow-up. Upon questioning, her husband reports a progressive decline in her mood and memory over the past week and you note the patient is exhibiting agitation and delusions during the consultation. She is re-admitted for further investigations and to re-commence IV aciclovir.
A lumbar puncture is performed and reveals a protein of 0.61g/dL, mild lymphocytic pleocytosis (17 per mm3) and negative HSV-1 PCR.
What is the most likely investigation to yield the correct diagnosis?
An 82-year-old patient with advanced stage Parkinson’s disease (PD) is referred for an inpatient neurology review. He has become confused following a recent change in his Parkinson’s medications. There are no other medications contributing to his symptoms and no acute medical concerns otherwise. Routine blood tests and a urine dip are normal with no evidence of infection.
You are asked to rationalise his medication to help mitigate these side effects. Which of the following is the first medication that you would consider stopping?
A 54-year-old lady presents to the emergency department with a 2-week history of left sided headaches, intermittent left ptosis and diplopia. She has a past medical history of obesity, type 2 diabetes mellitus and migraine.
On examination she had a complete left ptosis. There was abduction and depression of the left eye, mydriasis of the left pupil and a sluggish left pupillary response.
What is the most appropriate initial investigation?
A 65-year-old gentleman is seen in the Emergency Department with a progressive 2-week history of double vision and dysphagia. He has a past medical history of type 2 diabetes mellitus and hypertension. He has recently been started on clarithromycin for a chest infection.
His observations reveal a temperature of 37.6 degrees Celsius, heart rate of 105bpm, blood pressure 100/65mmHg, respiratory rate of 22/min and saturations 94% on room air. His examination reveals some proximal weakness with fatigability including fatigable ptosis on sustained upgaze. His neck flexion is 4/5 on the Modified Research Council (MRC) scale.
He is then referred to Neurology and has nerve conduction studies that show decrement and jitter consistent with myasthenia gravis. He starts treatment with pyridostigmine 30mg QDS.
Which of the following do not aggravate or induce myasthenia gravis?
Which of the following medications is a weak inhibitor of carbonic anhydrase?
A 30-year-old lady attends general neurology outpatient clinic with headaches. The diagnosis is consistent with tension type headache. However, she explains that her brother died recently of an aneurysmal subarachnoid haemorrhage (SAH) aged 40 and her mother died of a brain haemorrhage in her late 60s.
She has heard there can be a familial predisposition to aneurysms and asks to be screened.
Which of the following investigations is most appropriate?
A 62-year-old male has been admitted with a brief history of visual disturbance and headache shortly followed by confusion and seizures.
His examination reveals a GCS of 9 (M4V3E2) but he is maintaining his airway. He has a temperature of 37.6C, pulse 80bpm (regular), blood pressure 192/110mmHg, respiratory rate 18 and oxygen saturations of 88% on room air. A retinal examination reveals some copper wiring, cotton wool spots and A-V nipping without haemorrhages or papilloedema.
An MRI scan is shown below.
Which medication can cause this condition?
A 28-year-old warehouse operative presented to the fracture clinic with recurrent painless right shoulder dislocations, right shoulder swelling and limited right shoulder abduction.
On questioning, he stated that he would only feel pain in his lower limbs when he fell as a child. He would never feel pain from injections in his arms. Three years prior to admission he presented to the burns unit with second degree burns to his right hand, having fallen asleep with his hand on a radiator.
He was otherwise fit and well and took no regular medications. He smoked 8 cigarettes a day, tended to binge drink and occasionally inhaled nitrous oxide balloons. He was adopted so did not know his family history.
On examination his cranial nerves were normal. There was evidence of significant right shoulder effusion. He was unable to abduct his right shoulder beyond 15o. Power was otherwise normal in the upper limbs. Deep tendon reflexes were absent in the upper limbs. He had reduced pain and temperature sensation bilaterally in the upper limbs, down to a sensory level of T9, below which sensation was preserved. There was symmetrical hyperreflexia in the lower limbs.
What is the diagnosis?
A 24-year-old female presents with weakness and falls. She was quite clumsy as a child but her motor development was satisfactory. She was born by normal vaginal delivery with no complications. There is a similar presentation in her family; including her children (see figure below). Of note, the male at the top of the family tree is NOT a disease carrier.
On examination, you notice a foot drop with distal leg wasting and pes cavus. According to the MRC grading scale, ankle dorsiflexion power is 2/5, plantar flexion 4-/5, inversion 4/5, and eversion 3/5 bilaterally. Proximal motor exam was normal. Reflexes were absent throughout and sensation was reduced to the mid-shin to pinprick and light touch. Vibration was reduced to the tibial tuberosity and proprioception to the ankle. Upper limbs and cranial nerves were normal other than sensorineural deafness bilaterally requiring hearing aids.
Nerve conduction studies reveal intermediate conduction velocities, mildly prolonged F waves, and evidence of distal axonal loss. A cerebrospinal fluid examination is arranged which shows WCC=1/mm3, RCC=1/mm3, protein 0.8g/L, glucose 4 mmol/mol (plasma glucose 6).
What is the likely underlying condition?
A 12-year-old girl is brought to the Paediatric Neurology clinic by her parents. They describe recent daytime sleepiness, which is severe enough to disrupt schoolwork, for the past 6 months. More recently they have noticed episodes of slurred speech accompanied by a transient neck-drop.
The girl describes a constant sensation of fatigue. She says her nocturnal sleep is disrupted and, on several occasions, she has seen vivid images of people in her room on waking.
She reached her developmental milestones at normal ages. She had two febrile seizures at 1 year old but has otherwise been well with no neurological problems. A maternal aunt has multiple sclerosis.
On examination, she is normal height and weight. Cranial nerve and limb examinations are unremarkable. Cognition is grossly intact without formal assessment.
Which of the following tests would you choose to confirm your diagnosis?
A pregnant woman presents with a sudden onset right-sided headache, neck pain and contralateral weakness during the active phase of labour.
Examination reveals new unilateral weakness and a homonymous hemianopia. Blood pressure is 130/82mmHg, heart rate 92bpm, respiratory rate 19/min and temperature 37.2°C.
Which of the following is most likely to reveal the underlying cause?
A 38-year-old male with chronic cluster headache enquires about preventative medication.
Which of the following are potential candidates?
A 58-year-old male presented with a 4-day history of progressive and fluctuant bilateral lower limb weakness and paraesthesia with urinary retention. Past medical history was notable only for hypertension. His condition appeared to deteriorate following intravenous corticosteroids.
He underwent an MRI spine – cervical and upper thoracic sagittal imaging is shown below.
Which of the following investigations is most likely to confirm the diagnosis?
A 62-year-old gentleman presents to the outpatient Neurology department with bilateral hand tremors. His past medical history includes asthma and chronic migraine.
Regular medications are ventolin, beclomethasone inhaler, and amitriptyline.
On examination, you note a rhythmic 4-12Hz tremor of the outstretched hands on action and during isometric hold but not during rest. Archimedes spirals and handwriting are not micrographic. There is no evidence of an extrapyramidal disorder.
Which of the following medications would be optimal to start?
A 64-year-old man presents with a 3-year history of gradually progressive action tremor. The tremor has failed to respond to primidone and topiramate.
Past medical history includes mild depression and asthma.
Neurological examination is notable for an action tremor with intention component and a slightly broad-based gait. He undergoes MR imaging which is diagnostic.
Which of the following is most accurate in relation to this gentleman’s diagnosis?
A 70-year-old female has been admitted for an elective renal transplant for diabetic nephropathy. The donor is her younger sister. She has a past medical history of diabetes, chronic hypertension and obstructive sleep apnoea. Her medications include insulin, metformin, gliclazide, amlodipine and ramipril. She uses CPAP for sleep apnoea.
She is admitted to ICU following the transplant for monitoring, predominantly for blood pressure management and blood loss during the procedure. Two-days later she develops acute but fluctuating confusion, agitation, altered attention and tremor.
Observations reveal a heart rate of 90 beat per minute but otherwise are satisfactory.
An MRI-brain reveals cerebral atrophy and moderate small vessel disease. Her bloods reveal a mild increase in carbon dioxide but normal acid-base balance and a low bicarbonate. Full blood count and chemistries are normal.
What is the most likely cause for her presentation?
A 39-year-old woman has a 5-year history of deteriorating gait. Her paternal uncle passed away with a neurological disorder but she does not know the underlying diagnosis.
On examination, she has spasticity in the lower limbs with extensor plantar responses and clonus. Sensory examination is normal.
MR scans of brain and spine are normal. CSF examination reveals normal constituents with no oligoclonal bands.
What is the most likely diagnosis?
What is the most common cause of an oculomotor nerve palsy?
A 64-year-old female presents with acute onset visual changes. She has a past history of migraine and hypertension. She takes sumatriptan and amlodipine.
On examination, her GCS is 15 but when walking she is bumping in to objects and confabulates, describing objects in her visual field that do not exist. She fails to recognise the difference between light and dark when you test her pupil response with a pen torch. She fails to recognise simple objects you display, including a pen and a watch. She is unable to identify colours in a book and when talking to you her eyes fail to track your movements. Pupil responses, eye movements and fundoscopy are normal.
Her imaging is shown below.
What is the described phenomenon in this vignette?
You are analysing a local UK population of 10,000 and asked to estimate number of TIAs expected to occur within the catchment.
What is the closest estimate to this number?
A 56-year-old bus driver attends your clinic to assess his entitlement to renew his group II driving licence.
6 years previously he was in a road traffic accident. An MRI demonstrated a fractured calvarium overlying a small subdural haematoma with several small parenchymal contusions. His records state he was admitted to hospital and had two generalised tonic-clonic seizures in the early hours of the following morning.
The neurosurgeons decided to start levetiracetam. On outpatient review 6 months later, he was seizure-free. An EEG demonstrated no abnormalities. A repeat MRI showed complete resolution of the subarachnoid haemorrhage and focal areas of gliosis in the left cortex and juxtacortical regions. On this basis, the neurosurgeons discontinued antiepileptic treatment.
The patient tells you he has had no seizures since his admission 6 years ago and has not taken anti-epileptic medications since.
Neurological examination is normal.
Which of the following statements is true?
An 18-year-old barber presents to the Neurology Outpatient Clinic with a three-year history of progressive upper and lower limb weakness. He initially found walking upstairs and uphill difficult, which progressed to difficulty walking on flat and an inability to run. He now has to use his upper limbs to assist when standing from a sitting position and finds it difficult to keep his arms elevated when cutting clients’ hair.
He is otherwise fit and well. He participated in sports up until the age of 15. His father is estranged but is said to have walking difficulties.
On examination there is bilateral wasting of his deltoid and bicep muscles, with scapular winging. There is bilateral proximal upper limb weakness. Examination of the lower limbs demonstrated prominent wasting of the posterior thigh muscles and hip adductors. He was proximally weak in both lower limbs. Neurological and systemic examination were otherwise normal.
His ECG demonstrated normal sinus rhythm. His creatine kinase was >9000U/L.
Which of the following is the most likely diagnosis?
A 68-year-old gentleman with motor neuron disease (MND) presents to the MND clinic with a three-month history of dysphagia. He weighs 68kg, reduced from 76kg at his last review 2 months ago. His FVC is 1.9L (45% predicted).
He and his family maintain he has a reasonable quality of life. He does not wish to stop feeding but has choked several times and found the experience very distressing.
He is assessed by the speech and language team, who report severe oropharyngeal stage dysphagia. They conclude his swallow is unsafe for most consistencies.
What is the most appropriate intervention for feeding?
A 72-year-old man is admitted to hospital with spontaneous caecal volvulus. He undergoes an emergency hemicolectomy with a reversible colostomy and is put on bowel rest for 72 hours, after which the decision about feeding will be reviewed.
The surgical team contacts you for advice. The patient has long-standing generalised epilepsy, which is usually managed with carbamazepine 800mg twice daily and topiramate 250mg twice daily. They are concerned he is at risk of further seizures.
Concurrent medications include post-operative amoxicillin and meropenem, enalapril and metformin.
What is the most appropriate way to manage his epilepsy?
A 72-year-old gentleman presents to the Emergency Department via ambulance with acute-onset right-sided weakness.
He has an M1 thrombus on plain CT brain with associated territorial hypodensity. He was last seen well 5 hours ago.
His past medical history includes type 2 diabetes, hypercholesterolaemia and atrial fibrillation. His modified Rankin score is 3. Medications include metformin, dapagliflozin, atorvastatin and apixaban.
His National Institutes of Health Stroke Scale score (NIHSS) is 19 and the Alberta Stroke Program Early CT Score (ASPECTS) is 8. You have discussed the patient with the on-call interventional radiologist for consideration of thrombectomy.
What is the absolute contraindication to thrombectomy?
A 32-year-old male is admitted to the Emergency Department with weakness and sensory disturbance following a severe gastrointestinal illness involving vomiting, abdominal pain and diarrhoea, a few weeks earlier. He is a smelter by trade. He has no past medical history but did have to take nitrofurantoin for a recent urinary infection, which is being followed up accordingly.
On examination, he has symmetrical weakness 4/5 with diminished reflexes throughout. You also notice an additional feature highlighted in the figure below.
What treatment should be started?
An 18-year-old Caucasian boy presents with skin lesions and recurrent fevers.
On examination, he has aphthous ulcers, a right lower motor neurone facial palsy and multi-directional nystagmus with evidence of limb intention tremor and dysmetria. His GCS is 14/15 (M5V5E4).
His MRI brain is shown below. A lumbar puncture is performed and a CSF reveals 45 white cells (100% lymphocytes), 2 red cells, protein 0.9g/L, glucose 4mmol/L (plasma 6mmol/L) and matched oligoclonal bands.
What is the likely cause of his presentation?
A 35-year-old male is seen in the outpatient clinic with abnormal movements and difficulties feeding. He has a past medical history of anxiety, paranoia and learning difficulties and takes risperidone 2mg once daily.
During conversation, you note some vocal tics (grasping, sighing, whistling), repetitive tongue protrusions, orofacial dyskinesias, dysarthria and chorea. On formal examination you note bradykinesia, rigidity and subtle impairment in vertical upgaze. He was also overly friendly and, at times, socially inappropriate during the consultation. He scored 24/30 in the MoCA with impaired executive function and memory.
Blood tests show normal liver function with others pending and an MRI-brain, performed previously, revealed caudate head atrophy and dilatation of the anterior horns of the lateral ventricles.
What is likely diagnosis?
A 28-year-old female attends neurology outpatients’ clinic following an episode of blurred vision in her left eye and colours appear washed out. Six months previously she reported tingling in her arms lasting two weeks following the birth of her child. 12 months previously she experienced a severe bout of vertigo lasting 10 days.
Her past medical history includes migraines for which she is taking propranolol.
Your examination confirms a left optic neuritis and an MRI-brain scan reveals 9 demyelinating plaques with one lesion enhancing following administration of contrast.
She would like to conceive in the next year or two. Which of the following disease modifying treatment would be most appropriate?
A 72-year-old woman is admitted with headache and confusion on a background of mild cognitive decline in the preceding 12 months. Her husband reports she has become acutely confused since her headache began. She had been previously well and took no regular medication.
On examination, she lacked insight into her condition and was indifferent to food and drink. There was a right homonymous hemianopia with evidence of additional Gerstmann syndrome.
An MRI-brain revealed a diffuse leukodystophy (L>R) with diffuse meningeal and patchy gyral cortical gadolinium enhancement with no evidence of an acute stroke. An extensive investigatory work-up revealed evidence of a CSF lymphocytic pleocytosis (12 per mm3) but autoantibodies and RT-QuIC were normal. Digital subtraction angiography revealed evidence of beading and stenosis. Given the diagnostic uncertainty, a brain biopsy was performed, which revealed widespread concentric eosinophilic wall thickening of the leptomeningeal and cortical blood vessels, surrounded by lympho-histiocytic inflammation with positive immunostaining for amyloid-β.
What is the diagnosis?
A 60-year-old male with idiopathic Parkinson’s disease diagnosed 5 years ago is being considered for deep brain stimulation.
Which of the following is a correct indication?
A 34-year-old female presents to the emergency department with confusion, sweating and rigidity. She has a past medical history of schizophrenia and has recently started risperidone.
Her examination reveals a resting heart rate of 130 beats per minute, blood pressure 160/110mmHg, temperature 38.1°C and respiratory rate of 22/minute. Her GCS is 7 (M3 V2 E2), she is rigid in all four limbs and is markedly diaphoretic. The intensivist doctors have been called to assist.
There is no note of drug intoxication – although a toxicology screen has been sent – and her bloods show a white cell count of 14×109/L and creatine kinase 1100 U/L but there is no clear infective source and a pregnancy test is negative.
You commence IV fluids and stop her anti-psychotic due to concern about neuroleptic malignant syndrome (NMS).
What is the next line of management for NMS?
A 60-year-old gentleman diagnosed with painful chronic inflammatory demyelinating polyneuropathy (CIDP) has failed two first-line treatments. He has proximal and distal weakness with a severe sensory ataxia.
His blood tests reveal an HbA1C of 41mmol/mol, TSH 0.5 (range: 0.27-4.2mU/L), no paraprotein on serum/urine immunofixation, 2g urinary protein/ 24 hours, positive p-ANCA but negative PR3 and MPO antibodies. He undergoes a renal biopsy, which reveal a membranous glomerulonephritis. Nerve conduction studies reveal reduced motor conduction velocity, prolonged distal motor latencies, decreased CMAP amplitudes and conduction block.
What is the most likely diagnosis in this case?
A 32-year-old male presents to the outpatient department with long-standing but progressive muscle cramps and stiffness. He has no past medical history, takes no regular medications beyond occasional ibuprofen and denies recreational drug use.
On closer questioning, he reports stiffness predominantly in his legs on prolonged rest or sitting (especially after activity) which improves with exercise. It is also affected by extremes in temperature. He is estranged from his immediate family and is, therefore, unaware of any relevant family history.
On examination, there is evidence of gastrocnemius muscle hypertrophy and myotonia which improves with repetition. There is no muscle weakness. The remainder of the neurological exam is normal, as is the cardiovascular exam and there are no abnormal facial phenotypic features. An EMG reveals evidence of myotonic discharges. His ECG is normal.
Which of the following is most likely?
A 13-year-old male is seen in the epilepsy clinic following a bout of seizures, with additional myoclonus and hyperekplexia. He has a history of gradually progressive visual loss and his dilated fundoscopic examination is shown in the figure below.
On examination, you note additional ataxia but there is no evidence of psychomotor or cognitive deterioration and there is no facial dysmorphia. Fundoscopic imaging is illustrated below.
Which of the following mutations are most likely?
A 53-year-old female presents with acute hemiplegia progressing to quadriplegia over hours. On assessment, she gives no verbal or gestural response to questions.
Bidirectional horizontal gaze palsy is noted on examination with retained vertical gaze movement and blinking.
Where is the likely anatomical location of the presumed vascular event?
What is the most common cause of a trochlear nerve palsy?
A 32-year-old man is suffering hand weakness following shoulder, neck and side pain which came on after he exhibited a flu-like illness. The pain has subsided but his weakness remains.
On examination, you note weakness in flexion of the thumb and index finger. Sensation is preserved on all surfaces of the hand.
Which nerve is affected here?
You are asked to provide a prognosis for a 65-year-old woman on ICU. She suffered a witnessed cardiac arrest in the community 4 days prior. A bystander commenced CPR almost immediately and resuscitation was continued by the paramedics on arrival, with an estimated 16 minutes between loss of output and return of spontaneous circulation.
Her initial rhythm, recorded by the paramedics at 5 minutes after collapse, was ventricular fibrillation and 3 shocks were administered.
When you examine her on ICU, she is intubated and ventilated. Her pupils are not responsive to light and she does not blink in response to corneal stimulation. Tone is increased in arms and legs and plantar responses are extensor. She has no response to supraorbital pressure.
You request electrophysiological studies. An EEG shows generalised suppression. Visual evoked potentials show no cortical response to chequerboard patterns and somatosensory evoked potentials show no N20 cortical response to median nerve stimulation. An MRI obtained the same day shows generalised oedema with blurring of the grey-white matter borders.
Which of the following combinations of features most accurately predicts poor outcome (death or persistent unconsciousness) in this patient?
A 28-year-old female presents to neurology outpatients with progressive painless visual decline. She has no relevant past medical history and reports a normal diet. There is no clear family history. On further questioning, her vision has deteriorated over the past year, beginning in her right eye and later involving her left. She denies headache or pulsatile tinnitus.
On examination, her BMI is 26 and blood pressure 135/86mmHg. Her optic discs are pale, visual acuity 20/200 with reduction in red-green colour vision and contrast sensitivity. The remainder of the neurological examination is normal.
Her Humphrey visual fields are shown below. Her retinal nerve fibre layer reveals temporal sectoral thinning. An MRI brain and venogram is normal. A CSF is pending.
What is the likely cause for her visual decline?
A three-year-old boy is brought to the neurology clinic following unexplained developmental delay and alterations in behaviour. He was born at term without complication. Early development was normal but slowed such that milestones were delayed from around 6 months old. He also developed sand-like crystal in his nappies. He was late sitting up and has great difficulty in crawling. He never progressed to walking. His speech is minimal.
Multiple involuntary muscle contractions are present include writhing motions, involuntary muscle contractions and arching of the spine. On examination, you note damage to the lips, tongue and fingertips. This has been investigated previously and thought to be self-injurious. There was evidence of spasticity, hyperreflexia and extensor plantars.
You are requesting a diagnostic genetic test.
A mutation in which gene fits best with these symptoms?
A 7-year-old boy is seen in the paediatric neurology clinic following abrupt onset of choreiform movements, obsessive compulsive behaviours and restricted food intake. Further questioning reveals recent anxiety, emotional lability, aggression and deterioration in school performance. Other than a recent sore throat, he has no past medical history, met his milestones sufficiently until recently, had no complications at birth and was doing well in school.
Routine blood tests (full blood count, blood film, liver and kidney function) are normal.
Steroids and antibiotics are initiated with plasma exchange, which helps with his symptoms.
What is the likely diagnosis?
Which of the following anti-seizure medications is considered most safe for use during pregnancy?
A 42-year-old man with secondary progressive MS is brought to Accident and Emergency by his carer. He was diagnosed with relapsing remitting MS 15 years previously and was initially treated with beta interferon, but high disease activity prompted a switch to natalizumab. This was discontinued 3 years ago due to high anti-JC virus antibody titres. He was then switched to ocrelizumab. His last infusion was 4 months ago. His last relapse was 5 years ago with a partial transverse myelitis.
His baseline EDSS is 6.5 and a carer visits three times a day to help with self-care and meal preparation. He has an indwelling catheter and a baclofen pump for severe lower limb spasticity.
Medications, other than those mentioned above, include gabapentin 600mg TDS for pain, amitriptyline 100mg ON for pain and low mood, and prophylactic nitrofurantoin for recurrent urinary tract infections.
His carer describes a 24-hour history of rapidly progressive confusion and fever. He has omitted his usual medications today.
On examination, he is disoriented with evidence of paranoid ideation. He is flushed and diaphoretic with a low-level tachycardia (120 bpm), hypertension (180/100 mmHg) and fever (39.2°C). His chest is clear and his abdomen is soft. There is no abdominal tenderness. Neurological examination reveals a GCS of 14 (E4V4M6) with tremor, severe spasticity in all limbs and hyperreflexia. He has a sensory level at T10.
Blood tests reveal a normal haemoglobin level with lymphopaenia and acute kidney injury with elevated creatine kinase.
What is the most important aspect of his immediate management?
A 46-year-old male presented to hospital with a depressed level of consciousness. His past medical history includes familial hypercholesterolaemia, vertigo and testicular cancer. He drinks alcohol most days. On discussion with his wife she has noted him complaining of unsteadiness over the past couple of weeks with tinnitus, hearing loss and memory concerns.
On examination, his GCS is 10 (M5V3E3) and his observations are stable. There is vertical skew and a painful stimulus evokes an uncoordinated limb localisation. His plantars are extensor and reflexes brisk.
An MRI reveals a rhombencephalitis.
His LP reveals a moderate lymphocytic pleocytosis with gram stain and culture pending.
What is the likely cause?
A 26-year-old female, 6 weeks post-partum, presents with a third episode of sudden onset severe headache peaking within one minute. She had an uneventful delivery without complication. She has a past history of migraine controlled with sumatriptan when required and denies recreational drug use.
On examination, her GCS is 15 and she has no focal neurology with normal discs. Her urinary protein:creatinine ratio is 31mmol/mol and blood pressure 130/85mmHg.
Her imaging is shown below.
What is the likely cause for her presentation?
A study compared the incidence of lung cancer among smokers and non-smokers. Out of 1000 smokers, 30 developed lung cancer, while out of 1000 non-smokers, 10 developed lung cancer.
Calculate the risk ratio of lung cancer among smokers compared to non-smokers.
You are asked to review an 18-year-old’s epilepsy medication. He has macrocephaly. He has recently transitioned from the paediatric services.
What group of cortical developmental malformations does macrocephaly come under?
A 69-year-old female under investigation for cognitive decline by her GP for the last 2 months presents with dysphasia progressing over days to weeks.
She has difficulty following commands and is unable to provide any verbal output. She is generally quite limited in her movements globally. There are no lateralising neurological findings and she is afebrile.
An MRI is performed, with 2 axial DWI images shown below.
What is the most likely diagnosis?
A 78 year-old-female is referred to the outpatient clinic. She was diagnosed with Parkinson’s disease 18 months ago and currently takes co-careldopa 25/100mg TDS, which has been gradually up-titrated but unhelpful to date. She has had 4 completed falls in the last 6 months, resulting in head injury on one occasion. She describes some difficulty swallowing, a change in her voice and difficulty opening her eyes, particularly in the morning. She has a tendency to be emotionally labile, which is out of character.
On examination she has moderate dysarthria, slowing of vertical saccades, no tremor, moderate axial and symmetrical limb rigidity and moderate bradykinesia. Her gait is relatively upright and she takes 4 steps to correct her balance on a pull test. Her MRI is shown below.
What is the most likely diagnosis?
A 50-year-old female is seen in the outpatient neurology clinic due gait problems and “noisy” breathing at night. Her husband has a recording with noises heard on inspiration.
On examination, she has a broad-based gait with limb dysmetria and dysdiadochokinesia. Her speech is dysarthric, limb reflexes brisk with extensor plantar responses. There is bradykinesia with evidence of a jerky tremor of the outstretched hands.
An MRI-brain is performed and shown below.
What is the likely diagnosis?
A 36-year-old female is seen in the emergency department for a headache which has been persistent and pervasive for many weeks. She has no past medical history, denies recreational drugs, nasal sprays or supplements. She describes a sharp and stabbing pain that is very severe over the right temporal and supraorbital areas causing restlessness. Her right eye can droop and tear during the episode. It is occurring on average of 20 times per day and can lasts up to 30 minutes. Time to peak pain occurs in under 10 minutes.
In the emergency department she has a CT-brain, which is normal.
Her neurological examination is normal and Glasgow Coma Score (GCS) is 15.
What is the likely diagnosis?
A 34-year-old woman is referred to the headache clinic for management of migraine. Since the age of 16 she has suffered stereotyped unilateral, pounding headaches associated with photophobia, phonophobia and vomiting. There are no identifiable triggers and she has no aura. Headaches occur once and occasionally twice each week and last 12-24 hours. She is unable to work or partake in activities of daily life during headaches.
She uses sumatriptan for the most severe headaches, around twice per month, and high-dose aspirin typically once per week. She has tried amitriptyline and nortriptyline, neither of which were tolerated. Propranolol caused symptomatic postural hypotension.
Medical history is remarkable for gastro-oesophageal reflux disease, kidney stones and depression. Other than acute analgesia, she takes vitamin D supplements, omeprazole 20mg OD and sertraline 100mg OD.
Which of the following would you recommend?
An elderly female has been diagnosed with new-onset focal epilepsy and you are planning to start an anti-convulsant. She is taking warfarin for atrial fibrillation and has a history of previous psychotic depression.
With this information, what is the anti-seizure medication most likely to be beneficial with regard to adherence and seizure freedom rate?
Which of the following antiepileptics has not been associated with the development of chorea?
Which of the following signs is not seen in neuroleptic malignant syndrome?
A 21-year-old man returns to your clinic for review. Since the age of 16 he has suffered from a progressive condition characterised by gait ataxia, dysarthria and tremor. Over the intervening years he has developed epilepsy and now reports worsening muscle cramps and increasing cognitive difficulty, with family citing examples of forgetting important appointments and struggling to follow plots of television dramas.
On examination he has mild appendicular ataxia and dysarthria. Proximal lower limb power is reduced (hip flexion 4/5, hip extension 4+/5) with subtle fasciculations in the lower limbs and a spastic catch in both arms. Reflexes are brisk throughout and plantar reflexes are extensor.
Both parents and an older sister are well.
Routine blood tests (full blood count, renal function, liver function, calcium and CRP) are normal.
Mutations in the gene encoding what enzyme are most likely to account for his symptoms?
A 60-year-old female presents to the Emergency Department with intermittent and recurrent throat and neck pain with associated dysphagia.
On further questioning she reports pain around the left-side of the neck radiating into the left ear, ipsilateral tongue, tonsil and throat. This was not responding to NSAIDs and paracetamol, precipitating admission. There was no relevant history and no history of weight loss or B symptoms.
A CT-brain and CT-neck revealed a left-sided elongated styloid process.
What is the unifying diagnosis?
A 64-year-old right-handed male unexpectedly attends the Emergency Department with FAST positive symptoms. He works as an accountant and has a background of type 2 diabetes mellitus. He has a 20-pack year smoking history.
Further questioning reveals that he has been unable to calculate basic sums at work today and examination reveals some inability to co-ordinate his left and right.
An acute stroke is suspected and an urgent CT-brain is ordered whilst awaiting the stroke team’s arrival. Which area of the brain is most likely to be affected?
A 28-year-old man attends the outpatient department with abnormal nocturnal events occurring three times per night. As a child he was known to sleep-talk and exhibited regular awakenings but without diagnosis.
During the current episodes, his wife reports him sitting up, looking concerned and kicking his legs briefly. He appears afraid but does not respond when his wife tries to calm him. The events last around thirty seconds. You ask specifically about tongue biting or incontinence, which are absent. He has some memory of the events.
His examination reveals a BMI of 32kg/m2 but is otherwise normal. His Epworth sleepiness scale score is 12. He is otherwise healthy and denies recreational drug use.
What is the diagnosis?
A 59-year-old man is seen in the outpatient clinic. He suffered a transient loss of consciousness whilst driving 9 months previously on a hot summer day, resulting in a collision with another vehicle. There were no prodromal symptoms, no post ictal period and no witnessed convulsions (bystander history from passenger). He was reviewed in his local emergency department where there was no documented evidence of head injury and he had a normal ECG and CT brain. He subsequently went on to have a prolonged cardiac tape and echocardiogram on advice of the cardiology team. These have been reported as normal.
There is a 5-year history of Parkinson’s disease well-controlled with co-careldopa 25/100mg QDS. His license has been renewed the previous year with no concerns about driving safety. There is no history of epilepsy or previous syncopal episodes. At the time of your review there have been no further events. You are satisfied that the episode is compatible with isolated syncope.
The patient voluntarily surrendered his driving license after the event. He has now reapplied for his license (Group 1) and all relevant paperwork has been submitted to the DVLA. He has been informed of a potential several month back log with applications and asks if he can drive following his appointment with you.
Which of the following is most accurate with respect to the patients driving eligibility?
A 60-year-old gentleman is admitted with progressive proximal muscle weakness, dry mouth, erectile dysfunction and dizziness. On direct questioning, he reports some unintentional weight loss in recent times.
His observations reveal a temperature of 37.1°C, heart rate of 110bpm, blood pressure 90/62mmHg, respiratory rate of 18/min and saturations 98% on room air. His neurological examination reveals weakness of shoulder abduction 4-/5 which improves to 4+/5 after activity. He is also hyporeflexic throughout.
Which of the following tests, if positive, will support the likely diagnosis?
Which of the following features of meningioma is associated with a poor prognosis?
A 62-year-old male is admitted with acute onset diplopia.
On examination, you notice impairment of vertical gaze bilaterally and upon vertical upgaze there is some retraction of the globe and involuntary convergence. Voluntary convergence is impaired and the pupils respond minimally to light but are responsive during attempt at the accommodation reflex.
Where is the location of this lesion?
A 68-year-old male attends outpatients with dysesthesia and burning of moderate intensity in the region of his previous stroke. He suffered an acute stroke three months ago affecting the right arm and leg (see image below).
What is the name of this condition?
Which of the following medications is typically considered the most effective treatment for myoclonic epilepsy, provided there are no contraindications?
A 69-year-old female presents with new left-eyelid drooping, dysphonia, unsteadiness, self-rotary vertigo, nausea and vomiting. Her past medical history includes hypertension, hyperlipidaemia, Arnold-Chiari decompression and breast cancer in remission, the latter treated with a wide local excision and adjuvant radiotherapy.
On examination she has a left eyelid ptosis ~2mm with anisocoria and left eye miosis more marked in the dark. She also has ipsilateral ataxia, dysmetria and dysdiadochokinesia. There is additional diminished pain and temperature sensation over the left hemi-face and left limbs and trunk.
What is the most likely diagnosis?
An adolescent boy is seen in the sleep clinic for investigation following concerns from his mother regarding his sleep. He has bouts of excessive somnolence that can last from days to weeks, during which he sleeps 18-20 hours per day. During the episodes he only wakes for food and to void. He is also hypersexual, irritable with derealisation and confusion with childish behaviour when he wakes. There are no clear hypnogogic jerks or cataplexy and he is physiologically healthy and normal between bouts. There is no family history of neurological or sleep disorder.
His neurological examination is normal.
Brain imaging and lumbar punctures – including CSF orexin – are normal.
What is the diagnosis?
A 24-year-old Armenian male presents to the Emergency Department with a 3-day history of progressive bilateral leg weakness and sensory loss. He has a past medical history of asthma, fibromyalgia and recurrent orogenital ulcers.
On arrival he is febrile and unable to walk unaided. His upper limbs and cranial nerves are normal. Lower limb examination reveals symmetrical weakness of hip flexion (3/5), knee flexion (4-/5) and ankle dorsiflexion (4-/5). His knee jerks are brisk with spreading and ankle jerks brisk. Plantar responses are mute. There is reduced perception of pin prick to below the ribs, vibration sensation is lost to the anterior superior iliac spine, and joint position sensation is lost to the ankles. You notice papulopustular lesions on his legs and face.
His MRI reveals a longitudinally extensive myelopathy with a positive bagel sign.
What is the likely cause of his symptoms?
A 64-year-old woman is brought to Accident and Emergency by ambulance. She was seen well 8 hours ago by her son and was found 30 minutes ago by her friend with a right hemiplegia and expressive dysphasia.
On examination, she is normotensive and alert. NIHSS is shown below:
Item | Score |
1a (LOC responsiveness) | 0 |
1b (LOC questions) | 1 |
1c (LOC commands) | 0 |
2 (Eye movement) | 2 |
3 (Visual fields) | 2 |
4 (Facial palsy) | 1 |
5 (Motor arm) | 2 right, 0 left |
6 (Motor leg) | 2 right, 0 left |
7 (Limb ataxia) | 0 |
8 (Sensory) | 1 right, 0 left |
9 (Language) | 1 |
10 (Speech) | 1 |
11 (Inattention) | 0 |
Medical history includes well-controlled type 2 diabetes mellitus and hypertension.
A CT confirms a large left middle cerebral artery (MCA) infarct affecting approximately half of the artery’s territory. There is also a small infarct affecting approximately 20% of the territory of the ipsilateral anterior cerebral artery. CTA does not reveal any causative thrombus.
There are no contraindications, so she receives thrombolysis and is taken to the Stroke ward.
18 hours later she deteriorates. 1b on the NIHSS increases to 2 (unable to answer any question), 1c increases to 1 (performs only 1 of 2 tasks deteriorated). Facial palsy is more marked (2) and right arm is weaker (3). Language deteriorates to the point of incomprehensibility (2). She has an urgent CT scan, confirming worsening left hemisphere oedema.
Which of the following statements is true?
A patient’s brain biopsy is being discussed at the weekly neurosurgical multi-disciplinary team meeting. The neuropathology is shown below.
Which of the following is the most likely brain tumour in question?
Which of the following toxic compounds side effects are most likely to include abdominal pain, confusion, memory loss, and mood disorders?
You have received the genetics reports for a patient you suspect has Huntington’s disease. What is the minimum number of ‘CAG’ trinucleotide repeats that would definitely confirm your suspicion?
A 65-year-old gentleman has been admitted to hospital with intractable seizures. He has a background of a terminal glioblastoma multiforme. He has been given appropriate first-line therapy without success and the transition to palliation has been made with initiation of a midazolam continuous subcutaneous injection (CSCI). What is the recommended starting dose over 24 hours for an adult of average body weight?
Which statistical test should be used to assess the association between two categorical variables when the observed population n = 30?
A patient is being worked-up for an undifferentiated progressive sensory and motor peripheral neuropathy and undergoes a nerve biopsy (shown below).
Which of the following are associated with this appearance?
A 66-year-old female presents with a decline in walking speed and reports having generally “slowed down”. She has recently undergone a CT brain arranged by her general practitioner.
Which of the following clinical features would raise concern about proceeding to neurosurgical intervention in this case?
A 72-year-old female presents to the outpatient clinic with unsteadiness on her feet. She has noticed these symptoms after prolonged standing at church, which causes her legs to “freeze up” and give way.
What is the first line medication typically used to treat this condition?
A 64-year-old gentleman is seen in the Emergency Department due to progressive unremitting headache and papilloedema noted by his local optometrist, who advised admission.
He has no past medical history and is a non-smoker.
Neurological examination reveals some mild left-sided weakness and direct fundoscopy confirms the presence of optic disc swelling without haemorrhages.
An MRI-brain is shown below.
He undergoes a brain biopsy and the histology is reviewed by the neuropathologist, who confirms this is a WHO grade IV primary glioblastoma.
Which of the following altered tumour genomic/molecular profiles would support this diagnosis?
A 41-year-old South African male attended outpatient clinic with imbalance and cognitive symptoms, depression and irritability. He has a past medical history of diabetes and is maintained on metformin. He has a family history of a similar disorder in his mother and maternal grandfather without a clear diagnosis and passed away when he was young.
On examination, there was a generalised hyperkinetic movement disorder consistent with chorea and a rest tremor with bradykinesia in the upper limbs. Reflexes are brisk and cranial nerves reveal slowing saccades with some ocular dysmetria. There are no cerebellar signs. An Addenbrooke’s III score is 75 with deficits in a subcortical pattern.
An MRI-brain reveals atrophy of the caudate and cortex with brainstem and cerebellar sparing. Blood tests reveal a normal copper, caeruloplasmin and blood film. Genetic testing reveals a CTG trinucleotide repeat expansion of 43 triplets in the JPH3 gene.
What is the cause of this gentleman’s symptoms?
Which of the following autoantibody syndromes may cause a clinical presentation of diarrhoea, myoclonus, hyperekplexia, encephalopathy and weight loss?
A 23-year-old male presents to the emergency department with a one-week history of fever with later progressive confusion.
Neurological examination reveals weakness of the right arm and leg. There is no RAPD and the patient has normal optic discs.
A lumbar puncture reveals elevated protein and a mild pleocytosis without the presence of oligoclonal bands. No organisms are cultured and viral PCR is negative. MRI of the brain is shown below.
What is the most likely diagnosis in this patient?
Which of the following anti-seizure medications is least likely to worsen childhood absence epilepsy?
A 4-year-old boy presents with a history of sudden bouts of laughter and uncontrolled facial grimacing in a stereotyped fashion.
An EEG shows spikes and slow waves in the temporal region. An MRI-brain is shown below.
Which of the following is the most likely diagnosis?
A 28-year-old male with refractory idiopathic generalised epilepsy presents with worsening dizziness over the preceding few days. His medications include sodium valproate, lamotrigine and levetiracetam.
On examination, he has marked global ataxia and nystagmus.
Which medication interaction is likely the cause of his presentation?
A 45-year-old female is admitted with headache, fever and seizures. She is subsequently diagnosed with herpes simplex encephalitis.
Which main cognitive measure is most often affected in this condition?
A 72-year-old male presents to the Emergency Department with an acute onset of altered (warm) sensation and later numbness affecting right hemi-face and right upper limb. He has a past medical history of type 2 diabetes (recent HbA1c 72 mmol/mol), hypertension and depression. His symptoms started 48 hours previously and fluctuated over this time.
On examination, his NIHSS is 0 with a normal neurological examination. His ECG shows normal sinus rhythm.
His MRI (DWI) is shown below.
What is the diagnosis?
A 72-year-old gentleman is seen in cognitive clinic after his wife has noticed a progressive decline in his memory over the past 2 years. His symptoms have now progressed to the point where he requires help with certain activities of daily living.
On examination, he is afebrile, pulse is 42 beats/min, blood pressure 126/84mmHg, oxygen saturation 98% on room air and respiratory rate 16/min. There are no focal neurological deficits and no extrapyramidal signs. His mini-mental state examination reveals a score of 19, highlighting weaknesses in episodic anterograde memory, attention and visuospatial and executive domains.
An MRI reveals mesial-temporal and temporoparietal lobe atrophy without significant white matter changes. Cerebrospinal fluid reveals reduced amyloid-β 1–42/1–40 ratio (Aβ42/40), as well as increased p-tau and total tau.
Give the likely diagnosis, what medication could be started to help reduce the rate of decline in function?
A 74-year-old female was referred to The Outpatients Department’ following a 4-month history of unexplained anaesthesia across her left chin and lower lip. She has been seen by her dentist and ENT without an explainable cause. She has a past medical history of Paget’s disease, xerophthalmia and polymyalgia rheumatica requiring long-term steroids.
Examination reveals reduced sensation across the aforementioned area with no additional neurological abnormalities. The patient was edentulous.
What is the most likely cause?
A 7-year-old boy attends the Audiology department as his teachers and parents are concerned his hearing has deteriorated over the past 6 months. His teachers also report deterioration in behaviour, with frequent angry outbursts.
Audiogram is normal, although the audiologist noted difficulty following the commands, despite multiple repetitions.
3 months later, the boy is brought to the Emergency Department following a 2-minute episode of uncontrollable left arm jerking. On examination he is alert and observations are within the normal ranges. Cranial nerves are intact and reflexes, power and coordination are all normal. Speech is hesitant and somewhat slurred.
Blood tests including full blood count, electrolytes, liver function and glucose are normal. A CT head is unremarkable.
Which of the following would you be most likely to find on further testing?
Regarding chemotherapy-induced peripheral neuropathy, which of the following is incorrect?
A 62-year-old gentleman is being considered for a decompressive hemicraniectomy 52 hours following a proximal left M1 occlusion. He has a past medical history of diabetes, hypertension and is a lifelong smoker.
His National Institutes of Health Stroke Scale (NIHSS) score is 19, consciousness is decreased and he has evidence of an infarction involving 75% of the middle cerebral artery territory.
As per NICE guidance, which of the above are contraindications to decompressive hemicraniectomy?
The risk of converting from clinically isolated syndrome to MS (i.e., experiencing a relapse) is determined by 4 of the following.
Which of these options is NOT predictive of conversion to MS?
You are called to review a 40-year-old man on the Haematology ward. He was diagnosed with mantle cell lymphoma a year ago. Although it responded initially to a tyrosine kinase inhibitor, he had a radiological relapse 2 months ago and the decision was made to escalate to Chimeric Antigen Receptor T-cell (CAR-T-cell therapy). He received the cell infusion 7 days prior to your review.
The SHO explains they are concerned the patient has developed Immune Effector Cell-Associated Neurotoxicity Syndrome (ICANS) and would like your opinion.
On examination, the patient is sleepy but responsive to voice. There is no focal neurological deficit and optic discs appear normal.
His respiratory and cardiac observations are all within the normal ranges. Blood glucose is 5.1 mmol/L and he is apyrexial. CRP and interleukin-6 levels are checked daily and are normal.
You request an MRI and EEG. MRI brain is unremarkable. EEG shows diffuse slowing without epileptiform features.
What is your management?
A 9-year-old girl is seen in the Outpatients Department with progressive cognitive decline and balance issues. Her mother has noted a slow deterioration in her studies compared to her baseline. She has always required special-needs education. Of note, when she suffers from a cold or other viral-related illness her studies suffer more for a few weeks following recovery. In addition she suffers from poor balance on ambulation which has progressed over the past two years. Her past medical history includes delayed puberty, myopia and hypodontia.
On examination you note a short stature and gait ataxia. Limb examination reveal no motor or sensory deficit but significant intention tremor and dysmetria bilaterally. Her reflexes are brisk on both sides but her plantars went down. Her Montreal Cognitive Assessment was 26/30 with marks primarily deducted in executive function.
An MRI-brain is organised which revealed areas of hypomyelination with myelination present in the optic radiation. Moreover, a small T2 hypointense dot was observed in the posterior limb of the internal capsules and T2 hypointensities also seen in the venterolateral thalamus and dentate nucleus. There is also cerebellar atrophy and thinning of the corpus callosum.
In view of the imaging findings, a genetic leukodystrophy screen was sent which returned a heterozygous mutation in the POLR3B gene.
What is the unifying diagnosis?
A 12-month-old boy is brought to the outpatient Paediatric Neurology Department by his parents due to delayed acquisition of motor milestones. His parents report difficulty with head control and he is not crawling.
Examination reveals depressed reflexes, hypotonia and bilateral ptosis.
A muscle biopsy is shown in the figure below.
A mutation in what gene is most likely to be found in this patient?
A 29-year-old male undergoes a CT brain as part of a research study. This is reported to show bilateral calcification in the basal ganglia. She is referred to the neurology clinic for further evaluation.
She is estranged from her family but reports that her father developed tremor, slow unsteady walking and depression/anxiety in mid-life which got progressively worse. She denies any neurological symptoms and her only medical history is of mild anxiety and obsessive-compulsive disorder. There is no history of toxic/ chemical exposure and no regular medications.
Serum calcium, thyroid function and parathyroid hormone levels are all normal.
Which if the following is most accurate in relation to likely underlying condition?
A 31-year-old Japanese female presents to outpatient clinic with progressive 2-year history of unsteadiness. She has been using a stick to mobilise more recently.
On examination, you notice limb choreiform movements and evidence of limb rigidity, dysmetria, dysdiadochokinesia and heel-shin ataxia bilaterally. Cranial nerves revealed intersaccadic intrusions with multi-directional gaze-evoked nystagmus. MRI demonstrated cerebellar atrophy. A Montreal Cognitive Assessment score was 19/30 with delayed recall, attention and executive function most affected.
An Atrophin-1 gene mutation is identified.
What is the cause of her condition?
A 62-year-old man with Parkinson’s disease undergoes an MRI brain as part of work-up for deep brain stimulation. The scan identifies an asymptomatic incidental meningioma. On review of previous imaging this is noted on a CT brain 2 years previously and has not changed in size or appearance.
Which of the following is correct with respect to NICE guidance on radiological surveillance of this meningioma?
A study compared the prevalence of hypertension between two groups: a group of 200 people who drink coffee regularly and a group of 300 people who do not drink coffee. In the group of coffee drinkers, 60 people had hypertension, while in the group of non-coffee drinkers, 70 people had hypertension.
Calculate the odds ratio of hypertension in the group of coffee drinkers versus non-coffee drinkers.
A 32-year-old gentleman of Han Chinese origin with newly diagnosed epilepsy requires anti-convulsant treatment. HLA testing reveals the HLA-B*15:02 allele.
Which of the following is most strongly associated with the risk of a life-threatening adverse reaction in this patient?
A 74-year-old female with a background of ischaemic heart disease and migraine without aura presents with recurrent sensory symptoms spreading from the right hand to the elbow and then the face over minutes. This has happened four times without any impairment of consciousness, weakness or headache.
There are no other neurological findings and a CT-brain in the emergency department reveals a left-sided cortical subarachnoid haemorrhage (SAH). Her medications include clopidogrel, atorvastatin, bisoprolol and ramipril.
What is the next most helpful investigation for the probable diagnosis?
A 55-year-old male presents with sudden, painless vision loss in one eye.
Examination reveals optic disc swelling with a hyperaemic appearance with peripapillary splinter haemorrhages and evidence of a relative afferent pupillary defect. The contralateral eye has a small cup-to-disc ratio.
What is the most likely diagnosis?
A 66-year-old female seen in the outpatient clinic complains of a 6-month history of violent dream enactment and you suspect REM sleep behaviour disorder (RBD), which is subsequently confirmed with polysomnography. She has a past medical history of type 2 diabetes, depression, restless leg syndrome and mild depression.
Neurological examination is normal.
You suspect her RBD may be induced by a medication. Which of the following would be the most likely culprit?
A 67-year-old male is referred with difficulty standing from a sitting position. He also reports dizziness on standing, fatigue and dry mouth. He denies any sensory symptoms or sphincter disturbance. He is a current smoker and past medical history includes type 2 diabetes, hypertension and hypercholesterolaemia. Medications include atorvastatin, amlodipine, metformin, aspirin and insulin.
Examination revealed a supine blood pressure of 140/95mmHg (standing 110/75mmHg), heart rate 85 bpm, respiratory rate 14/min and temperature of 37.1°C. Cranial nerves were normal. Lower limb examination identified hip flexion power 4/5, hip extension 4+/5, knee extension 4+/5, knee flexion 5/5, dorsiflexion 5/5, plantar flexion 5/5 and hallux extension 5/5 bilaterally. Upper limb power was 5/5. Reflexes were diminished globally. Sensory and cerebellar examination was normal.
Investigations revealed a CK of 420U/L (normal range: 10-80U/L), LDH 160U/L (range: 140-280U/L), negative anti-nuclear, anti-acetylcholine receptor, anti-voltage gated calcium and paraneoplastic antibodies. An EMG shows facilitation at 50Hz stimulation, but not at 3Hz.
What is the most likely diagnosis?
A 53-year-old healthcare assistant presented to the emergency department with a 4-hour history of back pain and tetraparesis. Her symptoms started with numbness in her upper limbs and progressed to complete tetraparesis over 90 minutes.
On examination she had 3/5 power in the lower limbs with flaccid tone and reduced grip strength 3/5 in both hands. Reflexes were depressed in upper and lower limbs. Sensory examination revealed impaired appreciation of pin prick and light touch with a sensory level at C6
An MRI scan of her C-Spine was performed.
What is the diagnosis?
Which of the following is required as part of the diagnosis of brainstem death within the UK for adults?
Which of the following is most accurate in relation to the typical clinical manifestations of Huntington’s disease?
Which of the following drugs is not associated with a significant anticholinergic effect?
A 54-year-old female is admitted to the Emergency Department with fever, headache and confusion.
On examination, her temperature is 38.2, heart rate 78bpm, blood pressure 132/84mmHg, respiratory rate 14/minute and oxygen saturations 98% on room air. Neurological examination reveals a right relative afferent pupillary defect, bilateral intention tremors and dysmetria but normal limb tone, reflexes and sensation.
An MRI-brain is shown below. Cerebrospinal fluid reveals 80/mm3 white cells (80% lymphocytes), 2/mm3 red cells, protein 1.2g/L. Viral PCR is pending. Initial microscopy, culture and sensitivity is negative. Acid fast bacilli has returned negative. CT chest abdomen and pelvis in unremarkable.
Which of the following tests will help clarify the diagnosis?
A 26-year-old woman is referred to the Neurology clinic by her Ophthalmology consultant. Three months prior she had been diagnosed with unilateral optic neuritis. She denies any neurological symptoms before or since the event.
On examination, visual acuity is 6/4 on the right and 6/7.5 on the left, with a left RAPD and pallor of the left optic disc. The remainder of the examination is normal.
You organise further investigations. A lumbar puncture reveals acellular CSF with unpaired oligoclonal bands. An MRI brain and spinal cord shows two inflammatory-appearing T2 hyperintensities in the left frontal white matter with no enhancement after gadolinium.
What is the correct diagnosis?
Which of the following statements is correct with regard to febrile seizures in children?
Which of the following drugs may cause a myasthenic crisis in individuals with myasthenia gravis?
A 19-year-old Afro-Caribbean female presents to the Emergency Department with a 4-day history of paraesthesia in her lower limbs and unsteadiness on her feet. She studies dentistry at university and frequently attends socials. She takes the combined oral contraceptive pill and received a short course of nitrofurantoin 3 weeks ago for a urinary infection.
Examination reveals normal cranial nerves. Ankle jerks are absent but other deep tendon reflexes are present and symmetrical. Babinski’s sign is positive bilaterally. Romberg’s sign is present. Power of hip flexion is 4/5 on the MRC power scale but power is otherwise 5/5 throughout. Joint position sense is reduced to both ankles and vibration lost to the tibial tuberosities.
Blood work reveals Hb 120, MCV 102 fl, white cell count 9 x 10*9 cells, creatine kinase 62U/L(range: 40-320 U/L ), B12 180ng/ml (normal 200-900 ng/ml), folate 6ng/ml (normal: >4ng/ml), TSH 3mU/l (range: 0.38-5.3mU/l), HbA1c 30mmol/mol, zinc 12.3umol/l (10.1-20.2umol/l). Copper, homocysteine, methylmalonic acid and vitamin E are pending. An MRI whole spine reveals an inverted V sign on axial imaging.
What is the likely diagnosis?
You are asked to review a 42-year-old lady on the respiratory unit with right-sided blurred vision and diplopia. She has a past medical history of migraine, viral meningitis and a new lower respiratory tract infection and small pulmonary embolism. Her medications include topiramate, sumatriptan, IV amoxicillin and IV heparin.
On examination, she has a fever of 37.8°C, ptosis on the right with a near complete ophthalmoplegia. There is chemosis and proptosis but no orbital bruit. Visual acuity is 6/24 in the right eye. There is no fatigability. She looks as if she is in pain.
Her blood tests reveal a thrombocytopaenia.
What is the investigation that would most likely clinch the neurological diagnosis?
A 60-year-old female is referred to the outpatient clinic with progressive language impairment. There is a family history of motor neurone disease. During the consultation, you identify difficulties with word retrieval, noting frequent use of generic categories to illustrate the object in question. The patient seems to be relatively at ease with these deficits. Close family members have noted her experiencing problems with word meanings and she has been progressively more irritable and aggressive.
On examination, you note anomia, which does not improve with verbal cues. Surface dyslexia is evident with difficulty reading words such as “yacht”, “colonel”, and “debt”. Speech, grammar and repetition are spared and sentence comprehension is broadly better than single-word comprehension.
Limb examination reveals no frontal release or upper motor neurone signs. There is no ideomotor apraxia, parkinsonism, oculomotor abnormalities or gait impairment.
What is the likely cause?
A 52-year-old female with a background of hypertension, psoriasis and vestibular neuritis is admitted with reduced consciousness. Her medications include methotrexate, folic acid, amlodipine and PRN prochlorperazine. The evening prior to admission, she had been on a roller-coaster ride after having a few drinks. Her friends tell you she complained of spinning vertigo afterwards and had to take herself home. She is brought to the Emergency Department the next day with prolonged confusion.
On examination she is drowsy but rousable. Her GCS is 8 (M3 V2 E3). There is vertical skew deviation of gaze. Her limbs are moving but you are unable to test her power formally.
An MRI is performed and shown below.
What is the cause of her presentation?
A 62-year-old male presents to the cognitive clinic with progressive problems with speech. He has a past medical history of hypertension and hypercholesterolaemia. There is no family history of early dementia, psychosis, or motor neurone disease. He currently works from home as an accountant but is experiencing increasing difficulties with typing and writing.
On direct questioning, his wife reports he can drop words from sentences and use them out of order. His daughter had noticed reversal of binary terms, such as “yes” and “no” or “him” and “her” and difficulty in comprehension of complex sentences.
On examination, he exhibits anomia and significant agrammatism. Repetition, word meaning, semantic association and writing of irregularly spelled words are relatively spared. A Montreal Cognitive Assessment (MoCA) is 22/30 with impairment of executive function, fluency and visuospatial tasks. The remainder of the neurological examination is normal without frontal release signs.
An MRI-brain reveals dominant atrophy of Broca areas 44 and 45.
What is the probable diagnosis?
You see a 73-year-old female patient in the acute neurology clinic. She presented to the Emergency Department the day before with acute onset predominantly anterograde amnesia which had largely resolved by the time she was assessed 4 hours later. She was discharged with a provisional diagnosis of transient global amnesia.
Which of the following would lead you to consider an alternative diagnosis?
A 12-year-old girl attends the epilepsy clinic for annual follow-up.
From age 8 her parents noticed sudden, “jerky” movements, which could be provoked by sudden noise or sudden emotional changes. At age 9 she had her first generalised tonic-clonic seizure. These were initially well-controlled with lamotrigine, although she has had to increase her dose in the last year due to break-through generalised seizures. There is no family history of neurological disorders.
On examination, she is bright and alert with frequent proximal and distal myoclonic jerks that are exacerbated by sudden movements and noises. Cranial nerve examination is remarkable only for gaze-evoked nystagmus and mild dysarthria. Limb power, tone and sensation are normal but she has bilateral dysmetria and dysdiadochokinesis.
Full blood count, liver and kidney function are normal.
You diagnose a progressive myoclonic epilepsy (PME) and a genetic panel reveals a mutation in the gene encoding cystatin B.
Which of the following is true of her diagnosis?
A 13-year-old boy is seen in the epilepsy clinic following stimulus-sensitive myoclonus, increasing in frequency over 2 years. Over the past month he has had five generalised tonic clonic seizures and frequent falls.
He has ataxia on examination.
An MRI-brain is normal and an EEG reveals generalised spikes and waves.
There is no family history of epilepsy but genetic testing has revealed a mutation in the cystatin B gene.
Which of the following is the diagnosis?
A 23-year-old woman is brought to the Emergency Department with a debilitating headache. She describes a sudden onset occipital pain 45 minutes ago. On direct questioning, she is sensitive to light and nauseated but has not vomited. She denies head trauma or recreational drug use.
She has a history of depression and occasional sinusitis and attended hospital 6 weeks ago for a similar headache. She takes 100mg sertraline OD and oral decongestant PRN.
On examination, GCS is 15. She is clearly uncomfortable but is not meningitic. Cranial nerve, upper and lower limb examinations are normal.
An urgent CT scan is requested and is shown below.
Which of the following statements is true?
A 32-year-old female presents to the first fit clinic following a cluster of seizures which abated within 24 hours. She also presents a 2-year history of instability, falls, abnormal movements and progressive cognitive impairment. There is a family history of a similar presentation in her father and paternal grandmother but she is unaware of their diagnosis.
On examination you note some hyperkinetic limb movements consistent with chorea. She has a broad-based gait with appendicular ataxia. An Addenbrooke’s cognitive assessment revealed a score of 50/100, with deficits across all cognitive domains.
Her MRI revealed pontocerebellar atrophy.
A mutation in which of the following genes is the most likely cause?
A 46-year-old male presents with unsteadiness and seizures. He has a medical history of hypertension, autoimmune hypothyroidism, undifferentiated bone pain, hypercholesterolaemia and is being investigated for polydipsia and polyuria.
On examination you note bilateral (R>L) intention tremors with dysmetria and poor heel-shin co-ordination. There is also some subtle right-sided weakness – MRC 3/5 – in a pyramidal distribution. A MOCA is performed due to some subtle cognitive impairment noted subjectively, which reveals poor verbal fluency, executive function and memory.
An MRI-brain reveals an ill-defined tumour-like lesion around the pons and cerebellar peduncles with concomitant cerebellar atrophy.
A discussion with the neurosurgeons occurs and the decision to undergo a biopsy of the lesion is suggested. Neuropathology of the tissue reveals lipid-laden macrophages which stain positive for CD68, negative for CD1a and S100, with gliosis and Touton giant cell formation. Cells also stain for CD163 with surrounding demyelination. Molecular analysis reveals BRAF V600E variant.
What is the diagnosis?
The following screening tests and vaccination schedules are mandatory for which of the disease modifying therapies listed?
Which statistical test is appropriate to compare two means from independent samples which follow the normal distribution?
A 35-year-old woman is referred to the Emergency Neurology clinic by her GP. She reports a 3-day history of back pain, ascending numbness and lower limb weakness.
She has a history of Ro antibody-positive Sjogren’s disease and a provoked lower limb DVT following a Caesarean section 3 years prior. She is on vitamin B12 supplements after an incidental finding of macrocytic anaemia but no other medications.
On examination, cranial nerves are intact with normal visual acuity and healthy optic discs. Upper limb examination is remarkable for brisk reflexes and mild symmetrical weakness of finger flexion and abduction. Lower limb examination reveals a moderate spastic paraparesis with hyperreflexia and extensor plantar responses. She has a sensory level to pin-prick at the level of T2.
You arrange an urgent MRI spinal cord, shown below.
What is the most likely cause of her presentation?
A 58-year-old male is assessed in general neurology outpatients with a 1-year history of right-hand tremor and reduced finger dexterity. He reports stiffness of the right shoulder. Further questioning reveals sleep disturbance suggestive of REM sleep behaviour disorder and mild anxiety with no symptoms of autonomic disturbance or cognitive change.
Examination findings include reduced facial expression, normal speech, a unilateral thumb-involving rest tremor and mild-moderate cogwheel rigidity of the right upper limb. Gait assessment reveals a reduced arm swing on the right with otherwise normal initiation, speed and stride length and a negative pull test.
Which of the following is most accurate based on the above symptoms and signs?
A 64-year-old gentleman attends the Emergency Department with acute onset diplopia and imbalance. He has a past medical history of essential tremor and hypercholesterolaemia. His medications include primidone 250mg BD and atorvastatin 20mg ON.
On examination, his right eye is in the ‘down and out’ position with no anisocoria. There is also a left hand tremor and left hemiataxia.
Where is the lesion most likely to be?
A 52-year-old female passes away with rapidly progressive dementia. Her post-mortem brain biopsy reveals spongiform vacuolation throughout the grey matter, reactive astrocyte, microglial proliferation, neuronal loss and amyloid plaques.
What is her underlying neurological diagnosis?
A 74-year-old male presented to outpatient neurology with electric shock-like pains. He reports allodynia and dysesthesia in the right half of the mandibular region. He also reported ear pain deep in the right ear.
Examination revealed a right facial droop.
What is the likely diagnosis?
A 72-year-old man had noticed a 2-week history of a droopy right eyelid. His past medical history includes hypertension, iatrogenic hypothyroidism and type 2 diabetes mellitus. He was an ex-smoker of 20-pack-years.
On examination, he has right-sided ptosis and anisocoria, more pronounced in the dark, with the right pupil smaller than the left. Testing with 0.5% aproclonidine reversed the anisocoria but 1% topical hydroxyamphetamine did not cause pupillary dilatation.
Which of the following is the most likely cause of his signs?
A 22-year-old man attends clinic with his partner. Over the past 3 months he has woken her on 5 occasions by suddenly sitting up in bed, looking around in confusion, scratching his face and running his hands over the bedding repetitively as though searching and once picked up the alarm clock next to him. The first two instances she tried to wake him but he did not respond to her and returned to sleep. On the latter three occasions she has not tried to wake him and reports each episode lasted 3-5 minutes.
On direct questioning, she tells you he does not speak during the episodes. There is no posturing. The patient himself says he often frequently wakes feeling unrested but does not recall these episodes.
He has no medical history. Family history is remarkable for a maternal uncle with Parkinson’s disease. His father had similar episodes in his younger years.
Which of the following is the most appropriate treatment?
A 3-year-old boy is reviewed after his parents have noted multiple jerks, sudden falls and brief episodes during which he is unresponsive and appears vacant.
An EEG reveals bilateral synchronous 2- to 3-Hz spike and wave complexes on a 4- to 7-Hz background.
An MRI brain and routine bloods are unremarkable. Prior to the onset of these symptoms, he had been developing normally.
What is the likely diagnosis?
A 35-year-old female is reviewed in clinic with a progressive acroparaesthesia in her feet, as well as altered bowel habits. She is otherwise fit and well and takes no regular medications. Her father required a liver transplant and her paternal grandfather died unexpectedly at a young age.
On examination there is spinothalamic sensory lost to the mid-shins bilaterally. Her ankle jerks are absent but the remainder of the neurological examination was satisfactory. Her supine blood pressure was 124/78 (standing 92/60), heart rate 89 bpm, respiratory rate 14/minute and temperature 36.9°C.
What is the most likely diagnosis?
An EMG investigating the possibility of carpal tunnel syndrome reveals the following results.
Distal latency (t1)= 4ms
Proximal latency (t2) = 9ms
Distance between proximal to distal sites = 20cm
What is the conduction velocity?
A 60-year-old female is admitted to the Emergency Department with a sudden onset weakness of the right-hand. She has a past medical history of rheumatoid arthritis.
Her observations reveal a temperature of 37°C, heart rate of 85bpm, blood pressure 137/92mmHg, respiratory rate of 18/min and saturations 98% on room air. On examination, you notice her wrist extensors on the right are weak with 2/5 on the MRC power scale, reducing further on attempted fist closure. Her elbow extension power is 5/5, her reflexes are normal and sensation is reduced around the first dorsal interossei.
Where is the lesion likely to be within the neuroaxis?
A 53-year-old gentleman presents to the Neurology Outpatient Department with a 2-year history of progressive lower limb weakness and sensory symptoms. He initially noticed difficulty running, stating his legs felt weak and unstable after exercise. He also describes intermittent back pain, radiating into his right buttock. His buttocks are numb, and he reports urinary urgency and hesitancy.
At 18 years of age, he fractured two vertebrae playing rugby but did not require surgery.
On examination there is evidence of bilateral lower limb weakness, MRC power grade 4/5 for hip flexion and knee flexion bilaterally. Upper limb strength is normal. There is widespread hyperreflexia in the upper and lower limbs, with bilateral ankle clonus. He has reduced distal vibration sense bilaterally in the lower limbs, with saddle anaesthesia to light touch and pain.
MRI of his spine is shown below.
What is the next most appropriate investigation?
A 42-year-old man attends the Accident and Emergency Department in acute urinary retention. You are asked to review the patient, as he reported a 2-day history of lower back pain and bilateral leg weakness.
Other than hypertension, medical history is unremarkable. He takes only ramipril. His father was diagnosed with primary progressive multiple sclerosis aged 50.
On examination, he is now catheterised. Lower limb tone is normal without ankle clonus and there is bilateral symmetrical leg weakness, MRC grade 4 at the hip and knee and 3 at the ankle. Reflexes are absent and plantar responses are mute. There is a sensory level at the pelvis with saddle anaesthesia.
An MRI of the lumbar spine with gadolinium shows no compression but smooth enhancement of the cauda equina nerve roots. A CSF sample sent by the Acute Medical Team reveals a mild mixed pleocytosis (80% lymphocytes, 20% neutrophils) with a moderately increased protein concentration (0.92 g/dL) and a normal CSF:serum glucose ratio.
What of the following tests is most likely to be diagnostic?
Which of the following anti-seizure medications has the most significant impact on blood hormonal levels and may reduce the effectiveness of the combined oral contraceptive pill?
Regarding the pharmacokinetics of levodopa, which of the following statements is correct?
A 52-year-old male is admitted under the neurosurgeons with suspected cauda equina syndrome.
An urgent spinal MRI reveals a discontinuous T2 cord hyperintensity in the lower thoracic cord extending to the conus with smooth, linear nerve root enhancement but no surgical cauda equina syndrome.
He has no significant past medical history beyond recent genital ulcers. He has no mouth ulcers and a pathergy test is negative. There are no skin changes.
A CSF is examined which demonstrates a lymphocytic pleocytosis with no oligoclonal bands.
What is the most likely diagnosis?
Which of the following is a fully human antibody to the CD20 antigen that is used in the treatment of MS?
A 32-year-old female presents to the Emergency Department with sudden-onset severe headache and visual disturbances.
On examination, her heart rate is 72 beats/minute, blood pressure 82/54mmHg, temperature 37.2℃, oxygen saturations 98% on room air and respiratory rate is 12/minute. Neurological examination reveals constriction of both peripheral fields. The remainder of the neurological examination is normal.
Which of the following is the most likely diagnosis?
A study followed a cohort of 1000 people for 5 years and recorded the incidence of heart disease. Out of 500 men in the cohort, 100 developed heart disease, while out of 500 women, 50 developed heart disease.
Calculate the risk of heart disease in men and women.
A 20-month-old child with seizures and skin changes is being investigated at the regional paediatric neurology unit.
A CT-brain is performed (shown below).
In view of the scan findings, what is the most likely diagnosis?
A 32-year-old female presents to the Emergency Department with an acute drop in consciousness following a progressive 3-day headache history. She is 2 weeks post-partum from her first baby. She had three previous pregnancies unfortunately resulting in miscarriage. Her past medical history includes type 1 diabetes, for which she takes insulin.
On arrival, her GCS is 8 (M4V2E2) with bilateral extensor plantar responses and a livedoid rash. Her blood pressure is 140/72mmHg, heart rate 83 beats per minutes, temperature 37.1°C and respiratory rate 16/minute.
Initial bloods reveal platelets of 120,000/ul, haemoglobin 140g/L and white cell count 9,000/uL. Serum chemistries were satisfactory. Her CT-brain is unremarkable and her MRI-brain is shown below with diffusion weighted imaging sequence on the left and apparent diffusion coefficient sequence on the right.
What is the likely cause?
A 3-year-old boy is seen in the Neuropaediatrics Department following witnessed seizures. His parent’s report violent limb jerks lasting seconds, often in bright light, with preserved awareness but also describe events characterised by loss of consciousness with whole body shaking and head turning.
He was born without complication, albeit slightly small for gestational age. He has been otherwise well and reached developmental milestones at the appropriate ages.
On examination, he is short for his age and there is evidence of cerebellar ataxia.
Blood lactate levels and CK are raised and CSF examination is acellular but with a slight raised protein concentration at 0.5g/L and lactate 4mmol/L. An MRI brain reveals cerebellar atrophy and grey matter T2-weighted hyperintensities. EEG reveals a slow background with some generalised epileptiform discharges on photic stimulation.
A muscle biopsy is performed and shown below.
What is the most likely diagnosis?
A 42-year-old man presents to his local emergency department with 3 severe headaches in the past week. Today, the headache was accompanied by twitching of the right arm and leg and visual disturbance affecting his right hemifield. Twitching resolves with intravenous lorazepam and a mild left hemiparesis and visual deficits persist.
He has a history of oligodendroglioma treated with surgery and chemoradiotherapy 8 years previously. He had seizures at the time of diagnosis but none for 8 years. He takes levetiracetam 750mg BD but admits to poor compliance. CT angiography is normal.
MRI brain showed an area of cortical swelling and high T2 signal in the left occipital lobe extending to the adjacent parietal lobe, which enhanced with contrast (see below).
4 weeks later he continues to experience intermittent right sided twitching but headaches, visual disturbance and weakness have resolved. A repeat MRI demonstrates partial resolution of the changes (not shown).
Which of the following is the most likely diagnosis?
A 12-year-old girl attends your Neurology clinic with progressive unsteadiness, which presented at 5-years-old with clumsiness and falls. She has a history of bilateral toe clawing requiring bilateral cavovarus corrective surgery. There is no history of sensory symptoms, visual alteration or sphincter disturbance.
She is adopted and has no awareness of the health of her biological family history.
Examination reveals broken pursuit but with full range of eye movements and no nystagmus. She exhibits marked upper limb dysmetria and intention tremor and lower limb heel-shin ataxia, hammer toes, vibration loss to the ankles but with other sensory modalities intact. Her tone is spastic in her lower limbs but normal in the arms. Motor testing reveals mild global 4/5 weakness in her lower limbs only. Deep tendon reflexes are generally diminished or absent bilaterally and plantar responses are extensor.
Blood tests including vitamin B12, folate, thyroid function, vitamin E, phytanic acid, paraneoplastic antibodies, calcium channel antibodies, GAD antibodies, coeliac screen, lactate, copper, caeruloplasmin, cholestanol, and coenzyme Q10 are all negative or normal. An MRI-brain is shown below.
Nerve conduction studies (NCS) are arranged demonstrating prolonged distal latencies with decreased motor velocities in all limbs. Sensory NCS were unrecordable. In view of this presentation, fundal photographs and optic coherence tomography are performed which confirm retinal nerve fibre layer thickening.
What is the likely diagnosis?
A 29-year-old Cuban gentleman who is an office worker attends outpatient clinic with progressive walking difficulties over 8 years.
On examination, he has a broad-based gait. Ocular movements reveal saccadic slowing with an increased latency and saccade dysmetria. Pursuit it broken and the vestibulo-ocular reflex is impaired bilaterally. There is no nystagmus and eye movements are full. Limb examination reveals a rest tremor, bradykinesia and limb ataxia. Plantar responses are extensor.
An MRI scan is shown below.
What is the likely cause?
Which of the following drugs used to treat Parkinson’s is an antiviral?
A 32-year-old female with migraines is being reviewed in your outpatient Neurology clinic. She is currently on amitriptyline 40mg ON and has been for 6 months. Previous migraine preventatives include propranolol and candesartan, which were unsuccessful. She takes paracetamol 11-14 days of the month and triptans on 6 days.
She brings a headache diary with her. This records 8 migraine days and 17 headache days during the month. Her current treatment has provided a 20% reduction in her headaches.
Which of the following is the most appropriate next treatment option?
A 53-year-old man undergoes a CT head after a suspected first seizure (shown below). Past medical history includes anxiety and depression.
On examination you note mild symmetrical parkinsonism and mild gait ataxia.
CSF evaluation is unremarkable, including negative infectious serological work-up. A quantiferon test is outstanding.
Which of the following best explains these imaging abnormalities in this patient?
A 42-year-old female presents to outpatient clinic with jolt-like pains over the vertex. She reports pain predominantly on the right but the site can vary. The lancinating pains last for seconds and can occur one to multiple times per day. She denies any autonomic symptoms. She has no other past medical history.
Her brain imaging and neurological examination are normal. Her blood pressure is 135/85mmHg.
What is the likely diagnosis?
Which of the following toxins is commonly associated with the neurological symptoms of seizures, ataxia, hallucinations, and encephalopathy?
A 19-year-old male was referred for a neurosurgical opinion by his GP. He presented with a 3-day history of numbness and tingling in his feet that had ascended to his knees and he had difficulty passing urine.
He has no medical history and is otherwise well. He is on no regular medications. He drinks alcohol in moderation but on direct questioning admitted to marijuana and cocaine use.
On examination, power was 5/5 on the MRC scale in all limbs. Sensation to pinprick was intact, although vibration was not detected at the metatarsophalangeal joint. Bloods were unremarkable and an MRI of the lumbar spine was reported as normal.
He was discharged from the neurosurgical unit after passing urine. No diagnosis was recorded.
He has returned after 3 days with numbness and weakness in his hands. You are asked to provide an opinion.
On re-examination, gait is cautious. Cranial nerves are intact. Power is 5/5 on the MRC scale in the proximal arm muscles but 4/5 distally. Power in the proximal and distal leg muscle groups is 4/5. Biceps, triceps and supinator jerks are difficult to elicit and lower limb reflexes are absent with extensor plantar responses. Sensation is subjectively reduced to pinprick and vibration distally.
You request ANA, ANCA, HbA1C, thyroid function, B12, folate levels with Lyme, HIV and syphilis serology, all of which are normal/ negative. You also request an MRI of the cervical and thoracic spinal cord, shown below.
What is the test most likely to provide the underlying diagnosis?
A 35-year-old male presents with sudden onset atraumatic back pain, bilateral leg weakness, urinary retention, and saddle anaesthesia.
On examination there is 3/5 power in both lower limbs symmetrically with brisk knee reflexes but absent ankle jerks, extensor plantar responses and bilateral leg numbness.
What is the most likely diagnosis?
A 70-year-old male presents with progressive memory loss and difficulty with language.
A neurological examination reveals deficits in episodic memory and executive deficits. There are no motor deficits or abnormal movements.
Imaging studies show mild hippocampal atrophy. CSF reveals a normal amyloid-β 1–42/1–40 ratio (Aβ42/40), p-tau and total tau.
He later passes away and neuropathological evaluation shows TDP-43 protein inclusions in the hippocampal, amygdala and middle frontal gyrus.
Which of the following conditions is the most likely diagnosis?
A 76-year-old female attends the Emergency Department with sudden onset right facial droop and diplopia. She has a past medical history of left Bell’s palsy, pre-diabetes and left-eye esotropia in childhood.
On examination, you note a right facial droop without forehead sparing and with weakness of orbicularis oculi. On assessment of eye movements there is non-fatigable painless horizontal diplopia, an esotropic left squint and right eye abduction failure. Limb examination reveals no tremor, hemi-ataxia or choreoathetosis but there is some subtle weakness of the right-upper limb (MRC grade 4/5).
What is the most likely eponymous syndrome in question?
A 64-year-old female you have seen in clinic with mild cognitive impairment and a family history of dementia undergoes a lumbar puncture for cerebrospinal fluid neurodegenerative biomarkers.
Which of the following CSF profiles is the strongest predictor of developing Alzheimer’s dementia?
Which of the following may be compatible with a diagnosis of transient global amnesia (TGA)?
A 1-month-old female child is brought to the Emergency Department following a generalised tonic-clonic seizure. On questioning, the parents describe repeated clusters of events over the preceding week, during which the baby suddenly “stiffens up” for around 5 seconds. The baby was born at term by Caesarean section after foetal heart rate decelerations were detected by the cardiotocograph. Family history is unremarkable.
She is in the 50th centile for weight and height. She is apyrexial with no rashes. She shows abnormal stimulus-sensitive startle and multiple spasms are observed while she remains an inpatient, with 3 further generalised tonic clonic seizures over the first day.
Blood tests including full blood count, ammonia, calcium, magnesium, renal and liver functions and glucose are normal. CRP is slightly elevated. Ultrasound head shows no obvious intracranial abnormalities. Urine and blood cultures show no growth after 5 days. CSF analysis reveals a lymphocyte count of 3/microlitre, protein of 0.48 g/L, glucose of 3.2 mmol/mol (serum 5.4) and no organisms.
An EEG on day 2 of admission shows the pattern below.
Which of the following treatments is most likely to be successful in normalising the EEG and controlling seizures?
A 22-year-old female student presents with paraesthesia across the right medial forearm into the ring and little finger. She had been drinking heavily the night before. She had previously had an episode where she dragged her left foot for a period of days, which she noticed after spending a week in long lectures.
On presentation to the neurology clinic there is no focal neurological deficit.
An MRI-brain and cervical spine is normal. A tomaculous neuropathy is suspected.
What is the most likely cause?
Which of the following is a congenital myopathy that presents in early childhood with muscle weakness and wasting predominantly affecting the distal arms and legs, joint contractures (especially of the elbows and knees), and hypermobility of the hands and feet?
Which of the following medications predominantly blocks calcium channels as its mechanism of action?
A 43-year-old male is brought to hospital with an acute onset headache and an episode of vomiting.
On examination, he is alert and able to obey commands but is not oriented in place or time. Pupils are equal and reactive to light. He has a left upper motor neuron facial droop. There are no bulbar deficits. Right-sided limb power is 5/5 on the MRC scale but is 3/5 throughout on the left with a flaccid hemiparesis.
A CT scan confirms a subarachnoid haemorrhage.
According to the World Federation of Neurological Surgeons Subarachnoid Haemorrhage grading scale, what is the grade of the severity of the bleed?
According to the World Health Organisation, which one of the following is included in the clinical criteria of mild traumatic brain injury?
A 28-year-old male with a 2-year history of relapsing-remitting multiple sclerosis presents with symptoms compatible with optic neuritis. His estimated EDSS is 2.5. He receives a course of oral methylprednisolone with omeprazole cover.
One week later, he returns with left groin pain and hip flexor weakness. His vitals are stable and other systems examinations are satisfactory.
How should this be managed?
A 10-year-old boy presents with progressive visual impairment and cognitive decline. On examination, he has spasticity and hyperreflexia. An MRI-brain is shown in the figure below.
Genetic testing is most likely to reveal a mutation in which of the following genes?
You are asked to review a 25-year-old lady on the Obstetric ward. She had an elective Caesarean section 48 hours previously with a spinal anaesthetic and the procedure was uncomplicated.
Now she has regained full sensation in the lower limbs but has not been able to pass urine since removal of her catheter over 24 hours ago. A bladder scan revealed over 1L of urine in the bladder and an in-out catheter passed 6 hours ago relieved a volume of 1.2L.
She does not describe any pain, other than some soreness around the Caesarean incision site. She does not report any urge to pass urine
Medical history is remarkable only for infrequent migraine. She is taking 30mg dihydrocodeine TDS since the surgery but is otherwise on no regular medications.
On examination she has a palpable bladder. Upper limb examination is unremarkable. Power in the lower limbs is 5/5 on the MRC scale, although hip flexion is limited by pain. Reflexes are intact. There is subjective reduction in pin-prick sensation distally but no sensory level and a 512Hz tuning fork is detected at the metatarsophalangeal joint.
Which of the following investigations is most likely to confirm the underlying diagnosis?
A patient is admitted to the neurology ward with proximal lower limb weakness, ataxia, autonomic features. At rest he is areflexic but reflexes appear after isometric muscle contraction.
He is diagnosed with Lambert-Eaton myasthenic syndrome (LEMS), after which he receives acute immunotherapy and is started on 3,4-diaminopyridine.
What is the mechanism of action of 3,4-diaminopyridine?
Which of the following is correct in relation to Huntington’s disease (HD)?
Which of the following findings on FDG PET is most suggestive of dementia with Lewy bodies?
A 59-year-old gentleman attends the Neurology Outpatient Department with a progressive 2-year history of upper limb weakness. He has no significant past medical history.
On examination, you note symmetric wasting proximally and examination reveals weakness of shoulder abduction (MRC grade 2/5). Reflexes are brisk with a positive Hoffman’s sign. Spinothalamic and posterior column sensory testing is normal. Lower limb and cranial nerve examinations are normal.
EMG reveals large polyphasic potentials with fibrillations and positive sharp waves.
What is the likely cause?
A 52-year-old gentleman presents to the outpatient clinic with a progressive 3-year history of unsteadiness. He has a past medical history of hypertension and idiopathic peripheral neuropathy. His current medications include amlodipine, ramipril and over-the-counter medication for a chronic cough which started in his twenties. On closer questioning, he describes progressive imbalance, which is worse in the dark, progressive loss of feeling in his feet with paraesthesia and the sense that objects appear to oscillate when turning his head. He also reports dizziness on standing, erectile dysfunction and chronic constipation.
On examination, he had a broad-based gait with evidence of a length-dependent sensory neuropathy with absent ankle jerks but preserved muscle strength. Romberg’s test and head impulse test (bilaterally) were both positive. There was also evidence of intention tremor, dysmetria and dysdiadochokinesia. Eye movements revealed gaze-evoked horizontal and downbeat nystagmus with broken pursuit and dysmetric saccades.
An MRI-brain reveals cerebellar atrophy, predominantly of the vermis. Nerve conduction studies reveal reduced amplitude sensory action potentials with normal motor studies. Blink and masseter reflexes are impaired. An array of aetiological blood tests are negative or normal. You suspect a genetic cause.
Which of the following gene mutations is most likely?
A 7-year-old boy presents to the paediatric clinic with a history of unsteady gait, difficulty walking, and muscle weakness. The parents – who are of North-African descent – report that the child’s development was normal until the age of 5, when they started noticing that he had difficulty running and climbing stairs. He was also observed to have poor eye-hand coordination and unsteady movements.
On examination, the boy has a ‘no-no’ head tremor, bilateral dysmetria, a mild intention tremor, reduced muscle tone, depressed reflexes and decreased vibration sense with proprioceptive errors in his fingers.
Investigations reveal normal routine bloods, thyroid function, vitamin B12, phytanic acid, white cell enzymes and lipoprotein electrophoresis. An MRI brain reveals cerebellar atrophy. Nerve conductions studies are also normal but somatosensory evoked potentials are absent in the upper and lower limbs.
What is the most likely diagnosis?
A 3-year-old boy is seen in the paediatric neurology clinic for assessment of poor muscle control and intellectual disability. More recently his parents have noticed finger and lip biting, facial grimacing and abnormal movements. He is late to meet his motor milestones of sitting up and he is unable to walk or even crawl. He has also not developed his speech.
On examination, there is involuntary writhing consistent with choreoathetosis, facial grimacing and opisthotonous. Reflexes are brisk with extensor plantars bilaterally. There are bite marks around the fingers and lips. Abnormal sediment is noted within his nappy.
Which genetic mutation test would confirm this diagnosis?
A 74-year-old gentleman passed away with pneumonia following an undifferentiated neurodegenerative process. A section from the post-mortem histopathology is shown below.
What is the most likely underlying disease associated with this pathology?
Regarding device-assisted therapy for Parkinson’s disease (PD), which of the following is correct:
Regarding surgical treatment of idiopathic Parkinson’s disease (PD), which of the following is correct?
With respect to chorea, which of the following statements is correct?
A 72-year-old woman is referred to the Neurology clinic by her GP for management of severe facial pain. Five years ago she was diagnosed with left-sided trigeminal neuralgia, refractory to carbamazepine and lamotrigine and underwent balloon compression of the Gasserian ganglion, which was initially effective.
She describes 2 months of constant, burning pain with superimposed electric-shock-like sensations affecting her left temporal and periorbital region, exacerbated by even the most delicate cutaneous stimulation.
On examination, there is a region of exquisite sensitivity to fine touch above and around the left eye with patchy regions of numbness when the area is explored with a neurotip.
What is the likely diagnosis?
A 51-year-old male is referred to the general Outpatient Clinic with a persistent headache. He has experienced daily or near daily headache for the last 10 years. Originally, his headaches were episodic with severe unilateral orbital or maxillary pain lasting around 4 hours. Now, he describes a bilateral, severe throbbing pain, sometimes associated with nausea and photophobia. He experiences exacerbations of pain lasting several hours at least twice a week.
He is being treated by his GP for migraine and for the last few years has been established on a regime of pizotifen 1.5mg daily and he takes sumatriptan 50mg for each severe exacerbation.
On examination he has a normal systemic and neurological examination.
Regarding the initial evaluation of this patient, which of the following is most likely to be correct?
A 16-year-old child is transitioning to the adult neurology services with epilepsy secondary to periventricular nodular heterotopia (PNH).
What mutations are associated with classic PNH?
An 82-year-old gentleman with advanced Parkinson’s disease and associated dementia is seen in your Outpatient Clinic with his carer, who mentions he is struggling with troublesome urinary incontinence for which he has tried first line medication but with significant side effects. The patient is often unable to get to the toilet in time and it is becoming increasingly problematic.
Given his age and concomitant cognitive co-morbidity, which medication is most preferable?
A 42-year-old gentleman presents to The Emergency Department with new-onset focal seizures with secondary generalisation. His past medical history is unremarkable. He reports previous intravenous drug use but he has been clean for ‘many years’. His travel history includes a recent trip to China and Cambodia.
On your arrival he seems to have made a good recovery from his seizures and is conversational.
An MRI-brain is performed and shown below.
What is the diagnosis?
You are completing an afternoon ward round and review a 43-year-old man admitted to the Neurology ward with uncontrolled seizures following a road traffic accident 4 weeks prior. He was originally under the joint care of the Trauma & Orthopaedic Team and the neurosurgeons with a subdural haemorrhage, multiple cerebral contusions, spinal fractures at C6/7 and C7/8 resulting in cord compression with paraparesis and a fractured pelvis. His seizures have been controlled for the last week with 1.5g levetiracetam BD, 200mg lacosamide BD and 10mg clobazam ON.
The patient is in bed and reports a headache. He is alert and oriented but appears anxious. You check his current observations and note a heart rate of 42 bpm, a systolic blood pressure of 145mmHg and oxygen saturations of 98% with a mild tachypnoea. His face is flushed and sweaty. His abdomen is generally soft with a barely palpable bladder. You note he is catheterised and that he has widespread gooseflesh.
You perform a brief neurological examination. Pupils are miotic but reactive and the discs are normal. He has a full range of eye movements, no facial asymmetry and bulbar function is normal.
What is the next step in his management?
A 14-year-old female is seen in outpatient clinic with progressive weakness. She has a history of retinal vasculopathy and kyphoscoliosis. She takes no regular medications but admits to smoking ‘occasional’ marijuana. At school, she has struggled to keep up with her peers in physical education for many years.
On your examination, you notice she has a wide-eyed appearance, asymmetrical smile, sloped shoulders, scapular winging, prominent axial creases, asymmetric bicep atrophy and a protuberant abdomen. There are no visible contractures. Attempted shoulder abduction is slow and results in poly-hill sign. On sitting up, Beevor’s sign is positive. Eyelid closure is weak.
Power examination is detailed below (R;L):
Shoulder abduction 4/5; 4/5
Shoulder adduction 4+/5; 4/5
Elbow flexion 3/5; 4-/5
Elbow extension 3/5; 3/5
Wrist extension 3/5; 3/5
Wrist Flexion: 4/5; 4/5
Finger extension: 3/5; 3/5
Finger flexion: 4/5; 4+/5
First dorsal interossei: 4-/5; 4-/5
Abductor policis brevis: 4-/5; 4-/5
Abductor digiti minimi: 4/5; 4/5
Hip flexion: 3/5; 4+/5
Hip extension: 5/5; 5/5
Hip abduction: 5/5; 5/5
Hip adduction: 5/5; 5/5
Knee flexion: 3/5; 5/5
Knee extension: 3/5; 4+/5
Ankle dorsiflexion: 2/5; 4-/5
Ankle plantar flexion: 5/5; 5/5
Ankle inversion: 4/5; 5/5
Ankle eversion: 4+/5; 5/5
Hallux extension: 2/5; 2/5
Serum creatine kinase is 180U/L (normal 30-145U/L) and EMG reveals small polyphasic motor units with occasional fibrillations and positive sharp waves. Her ECG reveals normal sinus rhythm.
What is the likely diagnosis?
A 25-year-old male presents with a progressive difficulty walking and gradually worsening paraesthesia affecting his feet. There is a family history of similar foot symptoms.
On inspection, pes cavus is present and there is evidence of distal ankle weakness and loss of sensation to pinprick to the mid-shins bilaterally and vibration to the tibial tuberosity. Lower limb reflexes are absent.
Nerve conduction studies reveal conduction velocities of 32m/s in the tibial and 36m/s in the common peroneal nerves (normal values ≥ 38 m/s in both) with distal motor latencies of 7s between abductor hallucis and ankle (normal ≤ 6.5ms) and 7.2s between extensor digitorum brevis and the ankle (normal ≤ 6.5ms).
What is the most likely genetic mutation causing this presentation?
A 6-month-old boy presents with brief, sudden flexor or extensor movements that occur in clusters, often upon awakening.
His EEG is shown in the figure below.
Which of the following EEG patterns is demonstrated above?
A 50-year-old female attended alone for review in the cognitive disorders. She reports long-standing issues with memory, which have worsened in the last 2-3 years. She describes episodes of difficulty finding words and being unable to recall details of some recent family events. She was diagnosed with anxiety last year and takes Citalopram 10mg OD.
She lives with her husband and teenage son. She has worked as a secretary for over 20 years but has recently taken time off due to stress. She had a number of confrontations at work, which is unusual. Her mother was diagnosed with Alzheimer’s dementia at the age of 76.
She performs highly on neuropsychological testing, although displays relatively poor performance (5th centile) on an easy test of psychomotor speed and much better (60th centile) on a more challenging task. She self-rates her memory as low-average but performs above average in most domains of memory assessment.
She scores 13/21 on the anxiety subset and 8/21 on the depression subset of the Hospital Anxiety and Depression Scale. She has normal routine bloods and a normal MRI brain.
1 year later she reports persistent concerns about her memory. Repeat neuropsychology testing is stable.
What is the most likely diagnosis?
A 74-year-old male presents to the outpatient department with an insidiously progressive decline in language. He has a past history of dyslexia. During the consultation, you notice hesitations or false starts with circumlocutions.
On examination, he is unable to repeat sentences with a reduced verbal rate and phonologic paraphasias. Grammar, motor speech and single-word comprehension is intact. There is no ideomotor apraxia, parkinsonism, oculomotor abnormality or gait impairment.
An MRI brain reveals left temporo-parietal junction atrophy with thinning of the left superior temporal gyrus.
What is the likely cause of his presentation?
A 39-year-old woman with a background of controlled focal epilepsy attends outpatient clinic and enquires about contraception. She does not want to alter her anti-seizure medication and is suffering from no adverse side effects.
Which of the following is the most correct response with regard to contraception and carbamazepine?
A 58-year-old man is admitted with acute onset ataxia and vertigo. His symptoms improve over the course of 2 days.
An MRI brain with diffusion weighted imaging is negative for a stroke.
He tells you he has previous episodes that have presented similarly, one of which was diagnosed as a transient ischemic attack. He recalls his father having similar episodes.
He has a history of hypertension. He feels well between attacks.
Examination is notable for downbeat nystagmus and mild gait ataxia.
Reviewing the MRI scan with the neuroradiology consultant, subtle midline cerebellar atrophy is noted alongside mild-moderate small vessel ischemic disease.
What is the correct diagnosis?
A 70-year-old female with a previous diagnosis of restless leg syndrome attends the outpatient clinic. Her symptoms were well controlled with pramipexole MR for several years but have gradually returned. In the last 12 months she has noticed symptoms start earlier in the day and the uncomfortable sensation now involves her arms. Symptoms are temporarily relieved by moving but return shortly after. She reports being unable to sleep on several nights of the week due to her symptoms. Increasing the dose of pramipexole MR with her GP from 520mcg OD up to 1.05mg OD has not improved the situation.
Her neurological examination is normal. A recent serum ferritin is at the upper end of the normal range.
Which of the following would be the most appropriate next step in her management?
A 62-year-old male with Parkinson’s disease has attended the local neurology clinic after prompting by his wife due to sleep disturbances. He has no other past medical history and is self-caring. On closer questioning, his wife reports that he has been thrashing out in his sleep and that he occasionally shouts out at night. The patient is unaware of the events himself.
He is maintained on ropinirole. He denies taking any new medications and has been otherwise well in himself lately.
On examination, you note typical features of parkinsonism with no unexpected neurological findings.
What is next step in management to treat his symptoms?
A pregnant patient presents with pain, numbness, and tingling on the lateral side of the thigh.
On examination, there is numbness over the lateral thigh on the left. The remainder of the neurological examination is normal.
Which of the following is the most likely diagnosis?
A 26-year old lady presents to the emergency with a 4-week history of episodic severe headaches (2 per day lasting 1-2 hours). There is associated nasal lacrimation and tearing. An attack is aborted by oxygen therapy in the department.
She is 18-weeks pregnant, insulin-dependent diabetic and a current smoker. There is no history of headache disorder. Neurological examination is normal. A CT brain and intracranial venogram are normal.
A diagnosis of episodic cluster headache is made.
She has declined triptan therapy after discussing this with one of your colleagues but is keen to explore other options as symptoms are significantly impairing quality of life.
Which of the following is management option would you recommend?
A 32-year-old female presents with subacute encephalopathy and seizures leading to new-onset refractory status epilepticus. Her MRI brain is normal and lumbar puncture reveals a moderate lymphocytic pleocytosis with cultures pending. An EEG is shown below.
What is the likely test to confirm the probable diagnosis?
A 27-year-old Afro-Caribbean lady attends the emergency Neurology clinic with a 3-day history of ascending bilateral sensory disturbance affecting both legs and a 24-hour history of bilateral leg weakness and difficulty passing urine.
She has a history of systemic lupus erythematosus, diagnosed 3 years ago when investigation for bilateral small joint arthritis and a malar rash revealed elevated ANA and antibodies to dsDNA and cardiolipin. She has been asymptomatic on hydroxychloroquine 100mg BD. She also has Coeliac disease with anti-TTG antibodies.
A month prior to attendance she reports an episode of food poisoning with 2 weeks of constant vomiting and weight loss. Her mother also has SLE. Her father died of a stroke at age 62.
She takes hydroxychloroquine 100mg BD and aspirin 75mg OD.
She has not travelled abroad recently and does not smoke or use recreational drugs.
An MRI scan of her spinal cord revels a longitudinally extensive transverse myelitis.
What is the most likely diagnosis?
A 43-year-old French-Canadian female presents to the outpatient clinic with progressive eyelid drooping and dysphagia. She has no past medical history and takes no regular medications. Closer questioning reveals no diplopia and she denies any variation in symptoms. She thinks one of her siblings may have a similar condition but she was adopted and does not know an extensive family history.
Blood tests reveal a mildly raised creatine kinase 300UI/L (normal 100UI/L) with anti-acetylcholine receptor antibodies and anti-MuSK antibodies pending.
On examination, the patient exhibited an imperious look secondary to bilateral ptosis. Her extra-ocular movements revealed a complex ophthalmoplegia without fatigability and her swallowing time extended past 7 seconds when drinking 80mL of ice-cold water. There was no muscle weakness or fatigability.
EMG revealed no decrement and single fibre no jitter.
What is the likely cause of her symptoms?
Which heavy metal chelator is commonly used for the treatment of mercury poisoning?
Which of the following statements regarding treatment of status epilepticus in the landmark ESETT trial is true?
Which one of the following spinocerebellar ataxias (SCA) is associated with a pure cerebellar ataxia?
A 45-year-old male presents to the emergency department with hearing and balance issues. He reports his imbalance and vertigo can be induced by loud noises and his own speech is heard very loud in his right ear. He describes spinning vertigo and pulsatile tinnitus on the right.
Which of the following is the investigation most likely to secure the diagnosis?
A child is referred to your clinic by his GP for investigation of spastic paraparesis.
The 14-year-old boy attends with his father, who describes waking one morning 4 years ago and finding his son barely able to move his legs. There has been no recovery since onset.
The boy denies symptoms in his arms. Bowel and bladder function are normal. He has no medical history and takes no medications. His mother and maternal aunt have autoimmune thyroid disease but there is no family history of neurological disease. He denies recreational drug use. His family moved to the UK 4 months ago from Ethiopia.
On examination, cranial nerve and arm examinations are normal. The patient walks with a spastic gait and the aid of 2 crutches. There is wasting of the gluteal and quadriceps muscles. Both legs are spastic with >5 beats of clonus at each ankle. Power is symmetrically reduced in a pyramidal pattern (flexors 2-3/5 on the MRC power scale, extensors 3-4/5). Reflexes are brisk with extensor plantar responses. There is no sensory level to pinprick and he detects vibration at the metatarsophalangeal joint bilaterally. Temperature sensation is intact.
An MRI of his spinal cord is normal.
What is the most likely explanation for his symptoms?
A 19-year-old female attends the first fit clinic and you make a diagnosis of epilepsy. You are discussing management and seizure safety. The patient has read about SUDEP and asks for more information.
Which of the following is correct regarding sudden unexplained death in epilepsy (SUDEP)?